75 Days PDF (QUIZ) Template (8 TO 14) FINAL

You might also like

Download as docx, pdf, or txt
Download as docx, pdf, or txt
You are on page 1of 70

Day-8 Current MCQs:- Que-1 Consider the following statements: 1.

The United Nations General Assembly (UNGA) is one of the


six principal organs of the United Nations. 2. The UNGA appoints permanent members to the Security Council. 3. Majority of the
members in UNGA are developed nations. 4. The security council appoints the secretary general of the United Nations. Which
of the statements given above is/are NOT correct? a) 1 and 2 only b) 2 and 4 only c) 3 only d) 2, 3 and 4 Answer: D Solution:

 The UNGA appoints non-permanent members to the Security Council.

 Two thirds of the members in UNGA are developing nations.

 UNGA appoints the secretary general on the recommendation of Security Council. Enrich Your Learning: Membership and
participation in UNGA

 The United Nations General Assembly is one of the six principal organs of the United Nations and

the only one in which all member nations have equal representation.

 UNGA appoints non-permanent members to the Security Council.

 The General Assembly meets under its president or Secretary-General in regular yearly sessions

but It can also reconvene for special and emergency special sessions.

 It now has 193 members of which more than two-thirds are developing countries. Because of

their numbers, developing countries are often able to determine the agenda of the Assembly

(using coordinating groups like the G77), the character of its debates, and the nature of its

decisions.

 For many developing countries, the UN is the source of much of their diplomatic influence and

the principal outlet for their foreign relations initiatives.

 Africa – 55 seats

 Asia – 50 seats Appointment of secretary general of United Nations

 The UN General Assembly appoints the Secretary-General of the United Nations on

recommendation of the Security Council.

 The Secretary-General's selection is therefore subject to the veto of any of the five permanent

members of the Security Council.

 Mr. António Guterres, the ninth Secretary-General of the United Nations, took office on 1 January

2017.

 His predecessors were Ban Ki-moon from Korea (2007-2016), Kofi. A. Annan from Ghana (1997-

2006).

Que-2 Consider following statements with respect to Atal Bimit Vyakti Kalyan Yojna scheme: 1. Under this scheme, an insured
person is provided 60% of his average daily earning as relief. 2. It is paid for a maximum period of 180 days. 3. Considering rapid
employment changes, beneficiary is paid whenever he loses employment. Which of the above statements are INCORRECT? a) 1
and 2 only b) 2 and 3 only c) 1 and 3 only d) 1, 2 and 3 Answer: D Solution:

 Here an insured person will be provided 25% of his average daily earning as relief.

 It will be paid for a maximum period of 90 days.


 It will be paid once in lifetime of the insured person. Enrich Your Learning: What is it?

 It is a type of unemployment allowance.

 It seeks to support lost jobs caused due to changing employment patterns

 Here an insured person will be provided 25% of his average daily earning as relief.

 This will be based on previous four contribution periods.

 It will be paid for a maximum period of 90 days.

 And will be paid once in lifetime of the insured person. Who is implementing?

 Employees State Insurance Corporation under the Ministry of Labour and Employment. Likely benefits of the scheme?

 Provide monetary support in times of distress for an insured person.

 Create demand for employees to take insurance.

 The insured person will be eligible for Medical benefit Additional Details

 Insured person must fill a form in an affidavit.

 It will be paid electronically directly to the bank account of the beneficiary.

 The unemployment should not have been due to any punishment for misconduct, voluntary

retirement or superannuation.

 The beneficiary as to submit claims not later than one year from the date of unemployment.

 Periodical payments of Permanent Disability Benefit (PDB) under ESI Act and Regulations shall

continue.

Que-3 Larak Island was in news due to global geopolitics related to seizure of ship accused of smuggling oil. It is located in: a)
Strait of Bab-el-Mandeb b) Malacca Strait c) Strait of Hormuz d) Yucatan Strait Answer: C Solution & Enrich Your Learning:
Where is Larak Island?

 It’s an island in the Strait of Hormuz, under the jurisdiction of Iran.

Significance:

 It is one of the major export points of oil for Iran.


 Iran seized control of Panama flagged ship called MT Riah. Iran alleged that the ship was smuggling

oil. Strait of Bab-el-Mandeb:


Malacca Strait

Straits of Yucatan or Yucatan Channel

 In India the species is found in Tawang district of Arunachal Pradesh, Sikkim, Joshimath of

Uttarakhand, Lahaul and Spiti and Chamba districts of Himachal Pradesh. Enrich Your Learning: Why it was in News?

 Sikkim Knotweed has begun spreading along many streams and rivulets in the upper Nilgiris. It is

one of the species of invasive flora to pose a threat to native landscapes and biodiversity in the

Western Ghats.

 As the knotweed grows, it can cover huge expanses of land, leaving little to no space for other

native plant species to grow. About Polygonum molle:

 Polygonum molle, also known as Sikkim Knotweed, is an edible as well as medicinal herb.

 It is a perennial member in the family Polygonaceae. Its botanical name is Koenigia mollis.

 It can grow up to 1-3 m tall with large white flowers.

 In Nepal, the species has become endangered because of unsustainable extraction from the

forests for edible purpose as it has high demand in the market.

 The whole plant is astringent (capable of absorbing body fluids) and is used to treat diarrhoea.

The plant is also an efficient soil binder and covers the ground very well. Habitat
 The plant is found in forest, scrub and damp ground, slopes and in valleys.

 The species is distributed from Eastern Asia to Indian Himalayas and southwest China (i.e., Bhutan,

India, Indonesia, northern Myanmar, Nepal, Sikkim and Thailand).

 In India the species is found in Tawang district of Arunachal Pradesh, Sikkim, Joshimath of

Uttarakhand, Lahaul and Spiti and Chamba districts of Himachal Pradesh.

Que-5 Consider the following statements about Paramarsh scheme: 1. The scheme aims to improve quality of higher education
institutions who are ranked high in National Institutional Ranking Framework. 2. Hub and Spoke model is the mode of
implementation of this scheme. 3. The focus is on improving Government institutes only. Which of the above statements is/are
correct? a) Only 1 b) Only 3 c) 1, 2 and 3 d) Only 2 Answer: D Solution:

 Focus is on improving quality of non-accredited institutions to secure the National Assessment

and Accreditation Council grade.

 Mentor and Mentee institutes can be government or aided or private or self-financing institutes Enrich Your Learning: Why in
News? Recently UGC is expecting to receive 100 proposals under Paramarsh Scheme. What is it?

 It is a scheme to maintain and promote quality assurance in higher education.

 Here top 5 institution will mentor non-accredited institutions to secure the National Assessment

and Accreditation Council grade.

 It aims to have 1000 higher education institutes under this framework. Which Ministry?

 Under the Ministry of Human Resource Development.

 Implemented through University Grants Commission. What are its objectives?  It seeks improve the quality and standards of
higher education institutions.  Centralized control of operational efficiency and resource utilization of mentee colleges.  There
will be a Mentor Institute which will act as the Hub, while the Mentee institution would act as spoke.

Additional Details:  Mentor institutes must have NAAC grade of 3.26 out of 4  Both Mentor & Mentee institutes can be
government or aided or private or self-financing institutes.  UGC will monitor the progress by establishing Monitoring Centers.
 In the hub, external help from industry can also be sourced.  There will be a Mentor Institute which will act as the Hub, while
the Mentee institution would act as spoke. Benefits:

Que-6 Consider the following statements with reference to Regional Anti-Terrorist Structure (RATS) as sometimes seen in news:
1. It is a permanent organ of the Association of Southeast Asian Nations (ASEAN). 2. It is headquartered at Tashkent,
Uzbekistan. Which of the above statements is/are correct? a) Only 1 For Mentoring Institutes For Mentee Institutes For
Students

 Higher NIRF ranking  Increased collaboration  Regional institutional development.  Higher Quality education  Adopt best
practices  Improves reputation of the institution.  Boost to NIRF ranking  Better quality teaching and teachers.  Promote
Research collaboration  Provide quality education to nearly 3.6 crore students. b) Only 2 c) Both 1 and 2 d) Neither 1 nor 2
Answer: B Solution:

 It is a permanent organ of the Shanghai Cooperation Organisation (SCO). Enrich Your Learning: About Regional Anti-Terrorist
Structure (RATS) It is a permanent organ of the Shanghai Cooperation Organisation (SCO) which serves to promote cooperation
of member states against the terrorism, separatism and extremism. Its headquartered at Tashkent, Uzbekistan. The Head of
RATS is elected to a three-year term. Each member country of SCO also sends a permanent representative to RATS.

Que-7 Consider the following statements with reference to Pradhan Mantri Urja Ganga project: 1. It aims to provide piped
cooking (PNG) gas as well as CNG gas for the vehicles, to the residents of the eastern region of India. 2. It is implemented by the
Oil and Natural Gas Corporation (ONGC). 3. Under this project, the Government of India provides 40% capital grant for
implementation of this project. Which of the above statements is/are correct? a) 1 and 2 only b) 2 and 3 only c) 1 and 3 only d)
1, 2 and 3 Answer: C Solution:

 The project is being implemented by Gas Authority of India Limited (GAIL). About Pradhan Mantri Urja Ganga Project:

It is a gas pipeline project of government of India. This project, also known as Jagdishpur-Haldia & Bokaro- Dhamra Natural Gas
Pipeline (JHBDPL), was inaugurated in 2015. The overall objective of the project is to
 Phase-I: Pipeline segment from Phulpur near Allahabad in UP to Dhobi in Bihar.

 Phase-II: The pipeline will be extended to West Bengal and Odisha and connect via en-route spur
lines to Barauni, Haldia etc.

 Phase-III: The pipeline is extended to Assam via the Barauni-Guwahati pipeline by the end of 2021.

 Angul-Mumbai natural gas pipeline is another proposed extension on the JHBDPL pipeline system. Significance of the project:

 The project is considered as a major step towards collective growth and development of the

Eastern region of India. Under it, overall 20 lakh households will get PNG connections.

 It will usher Industrial development in East India by supplying environmentally clean natural gas

to fertiliser and power plant, refineries, steel plants and other industries.

 From Varanasi’s perspective, 50,000 households and 20,000 vehicles will get cleaner and cheaper

fuel PNG and CNG gas respectively.

 It will also help in revival of defunct fertilizer plants in Barauni in Bihar, Gorakhpur in UP, Sindri in

Jharkhand and Durgapur in West Bengal by supplying gas.

Que-8 In India, the number of Gharials have increased from 2012 to 2019. However, threats to their existence still remains.
Consider the following statements with reference to Gharials: 1. It is listed as Vulnerable in the International Union for
Conservation of Nature’s Red List. 2. In India, Chambal River has the largest population of Gharials in wild. 3. Gharials can be
found in Katerniaghat Wildlife Sanctuary. Which of the above statements is/are correct? a) 1 only b) 2 and 3 only c) 3 only d) 1,
2 and 3 Answer: B Solution:

 Gharials is listed as Critically Endangered in the International Union for Conservation of Nature’s

Red List. Enrich Your Learning: Gharial in India:

 When compared to alligators and crocodiles, a gharial has a very long and narrow snout (instead

of a broad snout). Habitat: They are a fresh-water crocodile. They live in deep fast-flowing rivers with high sand banks that they
use for basking and building nests. Gharial population is found only in India and Nepal. In India, they are found in Mahanadi
River, Girwa River, Son River, Chambal River, Ken River and Ramganga River. Major ‘breeding’ populations are confined to two
Chambal and Girwa rivers only. Chambal River supports largest population of Gharials in wild. Threat:

 They once inhabited all major river systems of Indian Subcontinent, from Irrawaddy River in east

to Indus River in west. Their total distribution is now limited to only 2% of their former range. IUCN Red List:

 Critically Endangered Threats:

 Being hunted for skins, trophies and indigenous medicine. Their eggs are also collected for

consumption.

 Major reason is decrease of riverine habitat as dams, barrages, irrigation canals and artificial

embankments were built; siltation and sand-mining changed river courses. Conservation & Protection:

 Species are Listed in Schedule 1 under Indian Wildlife Protection Act, 1972.

 Project Crocodile:

o It began in 1975 intensive captive breeding and rearing program.

o It was undertaken by Government of India in collaboration with United Nations

Development Fund (UNDF) & Food and Agriculture Organization (FAO) intensive captive
breeding and rearing program.

 Protected areas:

o National Chambal Sanctuary.

o Katerniaghat Wildlife Sanctuary. Que-9 Consider the following statements about Van Dhan Vikas Karyakram programme: 1.
Each Van Dhan Vikas Kendra supports five Self Van Dhan Vikas 'Samuh' (group). 2. Nodal Agency for implementation of the
scheme is Ministry of Tribal Affairs. Which of the above statements is/are correct? a) Only 1 b) Only 2 c) Both 1 and 2 d) Neither
1 nor 2 Answer: D Solution:  Each Van Dhan Vikas Kendra will support ten SHGs also called Van Dhan Vikas 'Samuh'.  Nodal
Agency for implementation of the scheme is TRIFED. While Ministry of Tribal Affairs is the

Nodal Department. Enrich Your Learning: Van Dhan Vikas Karyakram  This initiative comes under Van Dhan Yojana.  It seeks
to achieve two targets.

o Livelihood generation for tribal population

o Harness the wealth of forests Who is implementing?

National LevelNodal Department: Ministry of Tribal Affairs

Nodal Agency: TRIFED

State LevelNodal Agency: State Nodal Agency for MFPs

Local LevelNodal Agency: District Collectors

Van Dhan SHGsa Managing Committee from representatives of Van Dhan Self Help Groups (SHG)

What are the objectives?  In this scheme, traditional knowledge of tribal people and power of information technology (IT)

would be harmoniously utilized.  With a larger goal of making tribal wisdom a remunerative economic activity.  IT would be
used at each stage of the economic activity.  Van Dhan Vikas Kendras will provide skill upgradation and capacity building
training.  It will also act as primary processing and value addition facility for Self Help Groups (SHGs).  Van Dhan Vikas Kendra
would also act as a link with downstream processing centres. Likely benefits of the scheme?  It would promote economic
development of tribal population and the region they reside.  Support optimum utilization of natural resources.  Create
Minor Forest Produce based livelihoods. Additional Details

 Each Van Dhan Vikas Kendra will support ten SHGs also called Van Dhan Vikas 'Samuh'.

 Each SHGs will consist thirty tribal gatherers.

 Here SHGs can market their goods in the state as well as outside the state.

 State Government`s duty is to provide land to Van Dhan Vikas Kendras.


 There will be convergence of other government initiatives to ensure success of the scheme.

Que-10 Consider the following statements: 1. Genetic engineering is also known as Selective breeding. 2. Through Genetic
engineering, it is possible to extract DNA from an organism’s genome and insert it in another organism. Which of the above
statements is/are correct? a) Only 1 b) Only 2 c) Both 1 and 2 d) Neither 1 nor 2 Answer: B Solution: Genetic engineering is not
same as selective breeding.

Enrich Your Learning: Genetic Engineering

 Genetic engineering, sometimes called genetic modification, is the process of altering the DNA in

an organism’s genome. This may mean changing one base pair, removing a whole region of DNA,

or introducing an additional copy of a gene.

 Genetic engineering is used by scientists to enhance or modify the characteristics of an individual


organism. For example, genetic engineering can be used to produce plants that have a higher

nutritional value or can tolerate exposure to herbicides.

 Genetic engineering has a number of useful applications, including scientific research, agriculture
and technology. How does it work? Example of insulin: Insulin is a protein that helps regulate the sugar levels in our blood.
Normally insulin is produced in the pancreas, but in people with type 1 diabetes, have to inject insulin to control their blood
sugar levels. Genetic engineering has been used to produce a type of insulin from yeast and bacteria like E. coli.

A small piece of circular DNA called a plasmid is extracted from the bacteria or yeast cell. The gene for human insulin is inserted
into the gap in the plasmid. The genetically modified plasmid is introduced into a new bacteria or yeast cell. This cell then
divides rapidly and starts making insulin. To create large amounts of the cells, the genetically modified bacteria or yeast are
grown in large fermentation vessels that contain all the nutrients they need. When fermentation is complete, the mixture is
filtered to release the insulin.

Day-9 Current MCQs:- Que-1 Consider the following statements with reference to Reciprocal Trade Agreements (RTAs): 1. RTAs
are collection of only preferential trade arrangements that provide duty-free entry of several products. 2. Under RTAs,
members of World Trade Organization (WTO) can favour one trading country over another. Which of the above statements
is/arecorrect? a) Only 1 b) Only 2 c) Both 1 and 2 d) Neither 1 nor 2 Answer: B Solution:

 RTAs include many types of agreements, such as preferential arrangements, free trade

agreements, customs unions, and common markets in which members agree to open their

markets to each other’s exports by lowering trade barriers. Regional trade agreements (RTAs)

cover more than half of international trade today. Enrich Your Learning: Reciprocal Trade Agreements (RTAs):

 Countries use bilateral/regional trade agreements to increase market access and expand trade

in foreign markets. These agreements are called reciprocal trade agreements (RTAs).

 RTAs include many types of agreements, such as preferential arrangements, free trade

agreements, customs unions, and common markets in which members agree to open their

markets to each other’s exports by lowering trade barriers.

 Regional trade agreements (RTAs) are a key fixture in international trade relations.

 Over the years RTAs have not only increased in number but also in depth and complexity.

 WTO members and the Secretariat work to gather information and foster discussions on RTAs to

enhance transparency and to increase understanding of their impact on the wider multilateral

trading system. WTO rules on RTAs:

 Members of WTO can’t favour one trading partner over another but an exception to this rule is

RTAs.

 RTAs are discriminatory as only their signatories enjoy more favourable market-access

conditions.

 WTO members recognize the legitimate role of RTAs which aim at facilitating trade between its

parties but which do not raise trade barriers vis-à-vis third-parties.

 WTO members are permitted to enter into RTAs under specific rules like,

o The formation and operation of customs unions and free-trade areas,

o Regional or global arrangements for trade in goods between developing country

members.
Que-2 Consider the following statements about the Repurpose Used Cooking Oil (RUCO) programme: 1. RUCO aims to promote
the clean conversion of used cooking oil for further usage in cooking. 2. It is an initiative of Food Corporation of India. Which of
the above statements is/are correct? a) Only 1 b) Only 2 c) Both 1 and 2 d) Neither 1 nor 2 Answer: D Solution:

 Repurpose Used Cooking Oil (RUCO) is an ecosystem that enables the collection and conversion

of Used Cooking Oil (UCO) to biodiesel.

 RUCO is an initiative of The Food Safety and Standards Authority of India (FSSAI). Enrich Your Learning: Repurpose Used
Cooking Oil (RUCO) programme: Why in news? Recently, the Dehradun-based Indian Institute of Petroleum has successfully
finished a pilot test to convert used cooking oil into bio-aviation turbine fuel (Bio-ATF) which can be blended with conventional
ATF and used as aircraft fuel. About RUCO:

 Repurpose Used Cooking Oil (RUCO) is an initiative of The Food Safety and Standards Authority

of India (FSSAI).

 It is an ecosystem that will enable the collection and conversion of Used Cooking Oil (UCO) to

biodiesel.

 The chemical composition of the used cooking oil is identical to other plant-based oils that have

been converted to Bio-ATF such as Jatropha oil.

 Under this initiative, 64 companies at 101 locations have been identified to enable collection of

used cooking oil.

 According to FSSAI, it is possible to recover about 220 crore litres of used cooking oil for

conversion into bio-fuel by 2020 in India. Adverse effects of using Used cooking oil (UCO):

 The consumption of Used Cooking Oil(UCO) poses adverse health effects.

 It is associated with several diseases such as hypertension, atherosclerosis, Alzheimer’s disease,

liver diseases.

 Therefore, it is essential to monitor the quality of vegetable oils during frying.

Que-3 Consider the following statements with reference to the International Renewable Energy Agency (IRENA). 1.
Membership in IRENA is open only to regional intergovernmental economic-integration organisations of the world. 2. The Small
Island Developing States (SIDS) Lighthouses is an initiative of IRENA. 3. It has more than 150 member countries. Which of the
above statements is/are correct? a) 1 only b) 2 and 3 only c) 1 and 3 only d) 1, 2 and 3 Answer: B Solution:

 IRENA is not statutory body and it is an intergovernmental organization that supports countries

in their transition to a sustainable energy future.

 The IRENA membership in the agency is open to those states that are members of the United

Nations, and to regional intergovernmental economic-integration organisations.

Enrich Your Learning: International Renewable Energy Agency (IRENA):

 The International Renewable Energy Agency (IRENA) is an intergovernmental organization that

supports countries in their transition to a sustainable energy future.

 IRENA is an official United Nations observer.

 It serves as the principal platform for international co-operation, a center of excellence, and a
repository of policy, technology, resource and financial knowledge on renewable energy.

 In 2019, it has 160 members’ along with 23 countries in accession.

 India is a member of IRENA.

 IRENA Statute is available in in English and in four other United Nations official languages

(Arabic, French, Russian and Spanish) as well as in the language of the Depository (German).

 It is headquartered in Masdar City, Abu Dhabi. Background:

 The proposal for an international agency dedicated to renewable energy was made in 1981 at

the United Nations Conference on New and Renewable Sources of Energy, held in Nairobi,

Kenya.

 It was officially founded in Bonn, Germany in 2009 and its statute entered into force in 2010. Initiatives of IRENA:

 Clean Energy Corridors

 Coalition for Action

 Global Geothermal Alliance

 Parliamentary Network

 Renewable Energy Roadmap (REmap)

 Renewables Readiness Assessments

 The Small Island Developing States (SIDS) Lighthouses

 The Global Atlas for Renewable Energy Que-4 Oussudu lake, as recently seen in news, partly lies in which among the following
states? a) Tamil Nadu b) Odisha c) West Bengal d) Andhra Pradesh Answer: A Solution:

 Oussudu lake is located in Puducherry and the state of Tamil Nadu. Enrich Your Learning: Oussudu lake:

 Oussudu Lake is a lake which lies in the union territory of Puducherry and the state of Tamil

Nadu and it is located near the village named Oussudu. Key Facts:

 It has been declared significant wetlands in Asia by the Asian Wetland Bureau.

 It is the largest freshwater body in Puducherry.

 It is one of the major sanctuaries for migratory birds on the Coromandal Coast.

 The Oussudu lake is identified as a wetland of national importance under the National

Wetland Conservation Programme of the Ministry of Environment and Forests (MoEF).

Why in news?

 Recently, Indigenous Biodiversity Foundation (IBF-NGO) and Aaranyam (NGO) Foundation

have initiated steps to recreate habitats and artificial islands on the lake for migratory birds. Initiative of Artificial islands in
Oussudu:

 There are nine artificial islands and seven smaller mounds have been planned on the lake.

 The alluvial soil generated from desilting the lake bed will be formed into a mound for
creating the islands.

 The artificial islands to ensure a natural habitat with adequate prey for the birds.

 It also includes intensifying efforts to protect the flora and fauna of the lake.

 Migratory birds such as the Spot Billed Pelican, Painted Stork and Spoonbill Duck will return

once the islands are ready.

 The islands will create vegetation and foliage that will ensure the perfect habitat for nesting

and roosting of birds. Que-5 Consider the following statements with reference to the National Commission for Protection of
Child Rights (NCPCR). 1. It was set up under the Commissions for Protection of Child Rights (CPCR) Act, 2005. 2. NCPCR has
power to inquire into any matter which is pending before a State Commission for Protection of Child Rights. Which of the above
statements is/are Correct? a) Only 1 b) Only 2 c) Both 1 and 2 d) Neither 1 nor 2 Answer: A Solution:

 NCPCR is prohibited from inquiring into any matter which is pending before a State Commission

for Protection of Child Rights or any other Statutory Commission. Enrich Your Learning: The National Commission for Protection
of Child Rights (NCPCR):

 The National Commission for Protection of Child Rights (NCPCR) was set up in March 2007 under

the Commissions for Protection of Child Rights (CPCR) Act, 2005, an Act of Parliament

(December 2005).

 National Commission for Protection of Child Rights (NCPCR) is a statutory under the

administrative control of the Ministry of Women & Child Development.

 The Commission's Mandate is to ensure that all Laws, Policies, Programmes, and Administrative

Mechanisms are in consonance with the Child Rights perspective.

 The Child is defined as a person in the 0 to 18 years age group. Members of commission:

 A chairperson and six members (out of which at least two are woman) are appointed by the

Central Government. NCPCR Survey on MICA mine areas of Jharkhand and Bihar:

 The survey is about 'education & well-being of children in mica mining areas of Jharkhand and

Bihar'.

 India is one of the world's largest producers of mica, with Jharkhand and Bihar being the main

mica producing states in the country.

 Selling mica scraps is the main means of livelihood for many families in these areas. Highlights of the survey:

 Over 5,000 children in the age group of 6 to 14 years have abandoned education in mica mining

districts of Jharkhand and Bihar.

 Children have started working as labourers to supplement their family income.

 As per the survey, there are 4,545 children in the age group of six to 14 years in the area of

Jharkhand reported as not attending school.

 The reasons for not attending school included lack of aspiration, lack of interest and also cases
of collecting mica scraps, according to the survey.

Que-6 Consider the following statements with reference to recently launched India State of Forest Report: 1. 24% of India’s
geographical area is under forest and tree cover. 2. In India, the highest forest and tree cover is in Madhya Pradesh. Which of
the statements given above is/are correct? a) 1 only b) 2 only c) Both 1 and 2 d) Neither 1 nor 2 Answer: C Solution:Both the
statements are correct. Enrich Your Learning India State of Forest Report (ISFR) by Forest Survey of India

 The 24.49 per cent of India’s geographical area is under forest and tree cover.

 The Union Territory of Lakshadweep has highest forest cover (97%) as its geographical area is

only 30 square km.

 Among the six states with the highest forest and tree cover, four are in the Northeast –

Manipur, Arunachal Pradesh, Mizoram and Meghalaya.

 The lowest forest and tree cover in the country is in Haryana, at 6.79 per cent of its geographical

area.

 In terms of geographical area, the highest tree and forest cover is in Madhya Pradesh (85,487 sq

km).

 Goa and Kerala are two other states with more than 50 % of their geographical area under

forest and tree cover. Que-7 Which of the following Mobile app has been launched by the Ministry of Tourism through which
any Indian citizen can file complaints about unclean area around tourist destinations? a) Prawasi Pahera b) Saaf Mukam c)
Swachh Paryatan d) Sundar Smarak Answer: C Solution & Enrich Your Learning: Swachh Paryatan Mobile App Swachh
Paryatan Mobile App is operated by the Ministry of Tourism for 25 Adarsh Smarak Monuments. This mobile app is being
monitored by the Project Monitoring Unit of Swachh Bharat Mission in Ministry of Tourism. This mobile app enables a citizen to
take photograph of garbage at the monument and upload the same along with his/her remarks. The application then sends an
SMS to the ASI Nodal Officer concerned with the monument upon receipt of which the Nodal Officer gets the garbage
cleared/removed. The Ministry has launched an initiative for providing pre-loaded Sim Cards to foreign tourists arriving in India
on e-Visa. This initiative was launched in 2017 in association with Bharat Sanchar Nigam Ltd., (BSNL), wherein BSNL would
distribute pre-loaded SIM cards to foreign tourists on e-Visa. This initiative is aimed at providing connectivity to foreign tourists
to enable them to stay in touch with their family and friends and also help them to contact with the 24x7 multi lingual toll free
helpline of the Ministry for any assistance and guidance during times of distress/medical emergency, etc.

Que-8 Consider the following statements about National Agricultural Higher Education Project (NAHEP) and various agricultural
education research projects: 1. NAHEP is funded by the Indian Government and the Asian Development Bank on a 60:40 basis.
2. It is an initiative of Indian Council of Agricultural Research. Which of the above statements is/are correct? a) Only 1 b) Only 2
c) Both 1 and 2 d) Neither 1 nor 2 Answer: B Solution:

 NAHEP is funded by the Indian Government and the World Bank on a 50:50 basis. Enrich Your Learning: What is National
Agricultural Higher Education Project? A 1100 crores project launched by Indian Council of Agricultural Research. Funding
Funded on a 50:50 basis by the Indian Government and World Bank. Who are the Project Beneficiaries?

Other Measures Taken to boost agricultural education

 Declaring a four-year degree in Agriculture, Horticulture, Fisheries and Forestry as a professional

degree.

 Focus on experiential learning, skill and entrepreneurship development.

 Four new programs, B.Tech (Biotechnology), B.Sc. Community Science, B.Sc. Food Nutrition and

Dietetics and B.Sc. Sericulture have also been included.


 Upgradation of Rajendra Agriculture University has been upgraded as Dr. Rajendra Prasad

Central Agricultural University to strengthen the government’s effort to bring green revolution

in Eastern India including North East.

Students

-Shift to learning centric

education.

-Utilsing Information

Communication and

Technology interventions

-Better curricula development

-Greater equity in educational

access

-More vocational and technical

certificate porgrammes

Faculty

-Increased collaboration of

INdian Agriculture universities

with global universities

-Raise quality of research

-Improve educational quality

-Sharing of best practices from

the best in the world.

-Capacity building to imporve

delivery of education

Universities
-Better campus culture.

-Better governance

-Improve standard of

laboratories

-Improve physical infrastructure

 Indian Agricultural Research Institute (IARI) has been set up in Barhi, Jharkhand on the lines of

IARI, Pusa New Delhi, and another IARI is being set up in Assam.

 Promote agricultural business Student READY (Rural Entrepreneurship Awareness Development

Yojana) scheme is being run, where practical experience of agriculture and entrepreneurship will

be provided to undergraduate students.

 MoU between the Ministry of Agriculture and Farmers’ Welfare and the Ministry of Skill

Development and Entrepreneurship to promote skill development in the field of agriculture.

Krishi Vigyan Kendras will organise skill development programmes across the country.

Additional Information: 5thDean Committee Report: A committee of Vice Chancellors of Agriculture Universities, faculty
members, teachers as well as students Its objective

 To prepare an extensive report in this regard to make agricultural graduation education as

employment oriented,

 Rescheduling of the curriculum.

 Analyse assistance sought from the centre for strengthening agriculture education on state

level.

 To extend appropriate advice for the enrichment of professional knowledge and training to the

teachers related to agriculture.

 To issue necessary instructions for the improvement in administrative procedure in agriculture

universities and for the establishment of agriculture degree colleges. Outcome of the Committee: Created a manual

 for necessary administrative educational norms for the objective of their implementation and

 for the preparation of contemporary curriculum in respect to the agriculture related subjects as

agriculture, agriculture engineering, bio-technology, dairy technology, fisheries, horticulture and

sericulture.

Que-9 Consider the following statements about Fall Army Worm (FAW) invasive pest: 1. FAW originated from Africa. 2. The
larval stage of the FAW is the most damaging stage for crops. 3. FAW is a monophagous pest. Which of the above statements
is/are correct? a) 1 and 2 only b) 2 only c) 1 and 3 only d) 1, 2 and 3 Answer: B Solution: 1. FAW is an invasive pest with high
reproductive capability originating from South America. 3. FAW is a polyphagous pest i.e. it can feed on many types of food.
Enrich Your Learning: Why is in news? Recently, Union Ministry for Agriculture and Family Welfare in collaboration with
Safeguarding Agriculture and Farmers against Fall Armyworm and FMC Corporation launched a dedicated website regarding Fall
Army Worm (FAW). India had to import maize to support the poultry industry after FAW decimated maize crop. What is Fall
Army Worm?

 FAW is an invasive pest with high reproductive capability originating from South America and

spread all over the world, including India.

 In India, it first arrived in Karnataka and gradually has spread to Central, North and North East

India. It is also gradually spreading to other countries in Asia like Bangladesh, China, Indonesia,

Myanmar, Nepal etc.

 It can feed on many types of food like maize, sugarcane etc. Technically this ability is known as

polyphagous. Key traits of this worm

 FAW is a polyphagous pest i.e. it can feed on many types of food. Like maize plant,

bermudagrass, crabgrass, Digitaria spp, field maize, sweet corn and baby corn and alfalfa,

buckwheat, barley, maize, cotton, clover, millet, rice, maize, peanut, sorghum, ryegrass, Sudan

grass, Sugar beet and sugarcane etc.

 Can adapt to large number of host plants because its taste receptor genes can expand.

 The larval stage is the most damaging stage for crops.

 Colder temperatures reduce development of the larvae. But the ideal temperature for growth is

25-30 degree Celsius.

 Can migrate to fields during the night,

 Adult moths have strong flight abilities

 Pest can survive round the year. Impact of FAW

 FAW invasion can put world food security at risk.

 India had to import maize in order to meet the demand of the poultry and animal feed

industries after FAW destroyed the country’s maize crop in 2018.

 Result in huge losses to state exchequer

 Damage the supply chain of poultry and poultry feed industry.


 Trade and Transportation can facilitate global movement of FAW. Additional Information:

 The largest maize-producing state in India is Karnataka.

Que-10 Miyako Strait, as recently seen in news, is located in which of the following sea? a) South China Sea b) East China Sea c)
Yellow Sea d) Sea of Japan Answer: B Solution: Enrich Your Learning: Where is Miyako Strait located?

 A narrow passageway between Miyako Island and Okinawa Island

located in the East China Sea.

 This strait is the part of the Ryukyu/ Senkaku group of islands.

 This strait and group of islands are regularly mentioned in news. What is its significance?

 Territorial sovereignty issues and claims between Japan and China

over these islands.

 Japan calls these islands as Senkaku while China calls these as Diaoyu Islands.

 The 250 km strait provides the widest passageway for ships and aircrafts in the East China Sea.

 Controlling this area would be strategic for China as it can control a major chokepoint.

 There are frequent non adversarial skirmishes between China`s fishing and naval vessels and

Japan`s Coast Guard. Arguments:

Japan`s Argument China`s Argument

 Japan claims that in the 19th century,

nearly 200 nationals settled in these


islands to establish business of gathering

albatross feathers. These feathers are

used for making warm cloths.

 China started contesting in 1986, after a

UN Body citied possible oil reserves in

the area.

 China`s real objective is military in

nature.

 Before the Sino-Japan War, China and

Taiwan held the territory.

 Additionally, as a practice after the

second world war, where territories

would be returned to original owners.

China argues that Imperial Japan also

returned captured territories and should

also do the same in the present context.

Day-10 Current MCQs:- Que-1 Consider the following statements about Nuclear Energy Agency (NEA): 1. NEA was established
under the name of American Nuclear Energy Agency. 2. It works under the framework of Organization for Economic Co-
operation and Development (OECD). 3. NEA does not have any direct proliferation responsibility for nuclear explosive devices.
Which of the above statements is/are correct? a) 1 and 2 only b) 2 and 3 only c) 1 and 3 only d) 1, 2 and 3 Answer: B Solution: It
was established on 1 February 1958
under the name of the European Nuclear Energy Agency (ENEA), but to reflect its broader membership it was renamed as
Nuclear Energy Agency (NEA). Enrich Your Learning: Nuclear Energy Agency (NEA)

 The Nuclear Energy Agency (NEA) is an intergovernmental agency that facilitates co-operation among

countries with advanced nuclear technology infrastructures to seek excellence in nuclear safety,

technology, science, environment and law.

 It was established on 1 February 1958


under the name of the European Nuclear Energy Agency (ENEA)

but was renamed the Nuclear Energy Agency on 20 April 1972.

 NEA is under the framework of the Organization for Economic Co-operation and Development

(OECD), is headquartered in Paris, France.

 The NEA's current membership consists of 33 countriesin Europe, the Americas and the Asia-Pacific

region. India is not a member of NEA.

 Members of NEA together account for approximately 82% of the world's installed nuclear capacity.

One-fifth of the electricity produced from nuclear power in NEA member countries.
 NEA works closely with the International Atomic Energy Agency (IAEA) in Vienna and European

Commission in Brussels. Objective:  To assist member countries in maintaining and developing nuclear energy as a safe,
environmentally

acceptable, and economical energy source by serving as a forum where states can share information

and experience and promote international co-operation.  To prevent proliferation of nuclear explosive devices. However, the
Agency does not have direct

nonproliferation responsibilities. Areas of work:  Nuclear safety and regulation  Nuclear energy development  Radioactive
waste management  Radiological protection and public health  Nuclear law and liability  Nuclear science  Maintenance of a
data bank  Information and communication services Functions:  It maintains strategic partnerships with key non-member
countries involved in nuclear technology.  It establishes a climate of mutual trust and collaboration, enabling the full
exchange of experience

and a frank assessment of issues.  It publishes consensus positions on key issues, providing member countries with credible
references.  The NEA is cost-effective, relying on member country experts to carry out much of its technical work.  NEA
provides each member country access to the substantial experience of others and an opportunity

to substantially leverage its resources by using best nuclear expertise. Que-2 The ‘Copenhagen Criteria’ sometime seen in news
is: a) The rules for a country to join the European Union.

b) The guidelines for the reduction of Carbon emission.

c) The measures to be taken against the proliferation of nuclear arms. d) The eligibility criteria for European Free Trade
Association. Answer: A Solution:  The Copenhagen Criteria are the rules that define whether a country is eligible to join the
European

Union.  All the other options in the question is wrong. They do not relate to any of the logical statements. Enrich Your
Learning: Copenhagen Criteria

 The Copenhagen Criteria are the rules that define whether a country is eligible to join the European

Union.

 The criteria require that a state has the institutions to preserve democratic governance and human

rights.

 The other criteria include that the country has a functioning market economy and accepts the

obligations and intent of EU.

 These membership criteria were laid down at the June 1993 European Council in Copenhagen,

Denmark.

 For a country to be eligible to become a member of the European Union, the country is required to

be a European country.

 However, there are some international arrangements that allow such countries to integrate with the

European Union to some extent.

 One such arrangement allows non-European states to use the Euro as the official currency in their

respective economies.

Que-3 Consider the following statements about Hazardous and Other Wastes (Management& Transboundary Movement)
Amendment Rules, 2019: 1. Solid plastic waste cannot be imported into the Special Economic Zones of India. 2. Exporters of silk
waste have to ask for permission from the Ministry of Environment, Forest and Climate Change. Which of the above statements
is/are correct? a) Only 1 b) Only 2 c) Both 1 and 2 d) Neither 1 nor 2 Answer: A Solution:

 Exporters of silk waste have now been given exemption from requiring permission from the Ministry

of Environment, Forest and Climate Change. Enrich Your Learning: Hazardous and Other Wastes (Management&
Transboundary Movement) Amendment Rules, 2019 Recently, the Government has amended the Hazard Amendment in
Hazardous Waste (Management& Transboundary Movement) Rules, 2016 What is Hazardous Waste?  Any type of waste
which can cause danger to health or environment is Hazardous Waste.  The waste can be physical, chemical, biological,
reactive, toxic, flammable, corrosive or explosive in

nature. Hazardous Waste Management Rules 2016 Key Provisions:  Waste Management hierarchy in the sequence of priority
of prevention, minimization, reuse,

recycling, recovery, co-processing; and safe disposal has been incorporated.  Simplification of procedures through single
window clearance for setting up hazardous waste

disposal facility and import of other wastes.  Promotion of Co-processing facilities to dispose waste as well as generate
supplementary energy.  The import of metal scrap, paper waste and various categories of electrical and electronic

equipment’s for re-use purpose has been exempted from the need of obtaining Ministry’s

permission. Responsibilities of State Government for environmentally sound management of hazardous and other wastes
have been introduced as follows:  To set up/ allot industrial space or sheds for recycling, pre-processing and other utilization
of

hazardous or other waste.  To register the workers involved in recycling, pre-processing and other utilization activities.  To
form groups of workers to facilitate setting up such facilities.  To undertake industrial skill development activities and ensure
safety and health of workers.  List of processes generating hazardous wastes has been reviewed taking into account
technological

evolution in the industries.  List of Waste Constituents with Concentration Limits has been revised as per international
standard

and drinking water standard. The following items have been prohibited for import:  Waste edible fats and oil of animals, or
vegetable origin;  Household waste;  Critical Care Medical equipment;  Tyres for direct re-use purpose;  Solid Plastic wastes
including Pet bottles;  Waste electrical and electronic assemblies scrap;  Other chemical wastes especially in solvent form.
State Government is authorized to prepare integrated plan for effective implementation of these provisions and have to submit
annual report to Ministry of Environment, Forest and Climate Change. State Pollution Control Board (SPCB) is mandated to
prepare an annual inventory of the waste generated; waste recycled, recovered, utilised including co-processed; waste re-
exported and waste disposed and submit to the Central Pollution Control Board by the 30th day of September every year. What
does the amended rules aim for? To improve Ease of doing business and promote Make in India while ensuring environmental
sustainability.

Que-4 Consider the following statements with reference to India-UN Development Partnership Fund: 1. This fund is managed
by the United Nations Population Fund (UNFPA). 2. India is providing over INR 1000 crores to this fund over 10 years. 3. This
fund was established in 2017. Which of the above statements is/are correct? a) 1 only b) 2 and 3 only c) 1 and 3 only d) 1, 2 and
3 Answer: B Solution:

 This fund is managed by the UN Office for South-South Cooperation. Enrich Your Learning: India-UN Development
Partnership Fund Established in 2017. Key Features:

Salient features of the Hazardous and Other Wastes (Management& Transboundary Movement)

Amendment Rules, 2019

-Solid plastic waste cannot be imported into the country. Even in Special Economic Zones and
Export Oriented Units.

-Exporters of silk waste have now been given exemption from requiring permission from the

Ministry of Environment, Forest and Climate Change.

-Electrical and electronic assemblies and components manufactured in and exported from India, if

found defective can now be imported back into the country, within a year of export, without

obtaining permission from the Ministry of Environment, Forest and Climate Change.

-Industries which do not require consent under Water (Prevention and Control of Pollution) Act

1974 and Air (Prevention and Control of Pollution) Act 1981, are now exempted from requiring

authorization also under the Hazardous and Other Wastes (Management & Transboundary

Movement) Rules, 2016, provided that hazardous and other wastes generated by such industries

are handed over to the authorized actual users, waste collectors or disposal facilities.

 Supported by Government of India.

 Managed by UN Office for South-South Cooperation.

 Funds Southern-owned and led projects.

 Is Demand-driven.

 And Transformational sustainable development projects across the developing world.

 Focus on least developed countries, landlocked developing countries, Small Island Developing States

 Focus areas are in Africa, Asia and the Pacific, Latin America and the Caribbean, and Eastern Europe.

 India is providing $150 million funds over 10 years.

 A special mechanism to work with the Commonwealth developing countries. Significance:

 Improves India`s standing among the developing world and developed world.

 The Fund has developed a portfolio of 36 development projects in 37 developing countries.

 UN agencies implement these projects with governments of partner countries.

 Support Small Island Developing States building climate resilience and natural disaster preparedness.

 Promoting a collaborative development effort through South-South coordination.

 Help developing countries achieve Sustainable Development Goals.

Que-5 Recently a term ‘Okjokull’ was in news. The term refers to: a) A rejuvenated river b) A lost glacier c) A newly found island
d) A newly discovered animal species Answer: B Solution:

 Okjokull was a glacier in western Iceland who lost the status of glacier due to climate change. Enrich Your Learning: Okjokull
glacier

 Okjokull was a glacier in western Iceland on top of the volcanic mountain.


 This is the firsts glacier of Iceland to lose its status as a glacier due to climate change.

 The glacier was declared dead in 2014 and in 2019 a memorial plaque was installed to mourn its loss.

The plaque installation ceremony was attented by eminent personalities like PM Katrin Jakobsdottir,

Environment Minister Gudmundur Ingi Gudbrandsson and former Irish President Mary Robinson.

 Icelandic geologist Oddur Sigurðsson pronounced the Okjokull glacier extinct about a decade ago.

 The melted glacier was the subject of the 2018 documentary “Not Ok”. Que-6 Which of the following is India’s lightest
indigenously made Bulletproof jacket? a) Varuth Kavach b) Raksha Kavach c) Bhabha Kavach d) Vijay Kavach Answer: C Solution
& Enrich Your Learning: Bhabha Kavach

 It is India’s lightest indigenously made Bulletproof jacket.

 It is developed by Ordnance Factory Board (OFB) and Mishra Dhatu Nigam Limited (MIDHANI).

 It has 360 Degree Protection and has achieved the protection level of NIJ III+ (US National Institute of

Justice).

 It has embedded Nano technology from Bhabha Atomic Research Centre (BARC).

 It will be helpful for paramilitary forces and state police forces operating in Naxalite areas.

Que-7 Consider the following statements about International Finance Corporation (IFC): 1. IFC offers Investment services as well
as Advisory services to the developing countries. 2. It is a member of Organisation for Economic Co-operation and Development
(OECD). 3. The programme called EDGE (Excellence in Design for Greater Efficiencies) was initiated by the IFC. Which of the
above options is/are correct? a) 2 only b) 2 and 3 only c) 1 and 3 only d) 1 and 2 only Answer: C Solution:

 IFC is a member of World Bank. Enrich Your Learning: International Finance Corporation (IFC)

 A member of the World Bank Group

 Focuses on developing private sector in developing countries. Services offered:

 Investment services

 Advisory services and

 Asset management services Has Two goals:

 By 2030: end extreme poverty and

 promote shared prosperity in every country Aim is to advance economic development by investing for profit and commercial
projects for poverty reduction and promoting development. IFC is owned and governed by its member countries. Shareholders
are member governments and provide paid-in capital and right to vote on its matters. Features:

 Lead mobiliser of third part resources for projects

 Projects evaluated by the Independent Evaluation Group. Recent Initiatives:

 Created a mass-market certification system for fast growing emerging markets called EDGE

("Excellence in Design for Greater Efficiencies")

 IFC and the World Green Building Council have partnered to accelerate green building growth in less

developed counties

Que-8 Consider the following statements about Deep Ocean Mission: 1. The mission seek to address issues arising from long
term changes in the oceans due to climate change. 2. The Government has allocated 8000 crores to complete the mission. 3.
Department of Biotechnology is one of the lead agencies for implementation of this mission. Which of the above options is/are
correct? a) 1 only b) 2 and 3 only c) 1 and 3 only d) 1, 2 and 3 Answer: B Solution:  The mission does seek to address issues
arising from long term changes in the oceans due to climate

change.

Enrich Your Learning: Deep Ocean Mission  8000 crores and a Five-year mission.  Under the Union Earth Sciences Ministry.
Purpose  Explore the deep ocean for marine resources, living and non-living.  Address issues arising from long term changes
in the oceans due to climate change.  Development of underwater vehicles and underwater robotics, which can carry 3 people
and go at

least 6000 meters.  Development of ocean climate change advisory services  technological innovation and conservational
methods for sustainable utilization of marine bio-

resources;  offshore based desalination techniques;  and renewable energy generation. Other Facts:  From the UN
International Sea Bed Authority, India has been allotted a 75,000 sq. km area in the

Central Indian Ocean Basin to explore and exploit polymetallic nodules. Estimated to be 380 million

metric tonnes.  Polymetallic nodules are rocks contain iron, manganese, nickel and cobalt, found on the seabed.

Que-9 Consider the following statements with reference to Market Intervention Scheme (MIS): 1. The State government itself
implements the MIS scheme for a group of food products having high price fluctuation. 2. MIS is implemented when there is at
least 10% increase in production or 10% decrease in the prices over the previous normal year. Which of the above statements
is/are correct? a) Only 1 b) Only 2
 Jammu and Kashmir rolled out the 6-month Special Market Intervention Price Scheme (MISP) of

Government of India to procure nearly 12 Lakh Metric Tons of Apple from farmers of Kashmir on

remunerative prices.

 The nodal procurement agency from Government of India is National Agricultural Cooperative

Marketing Federation of India Ltd. (NAFED). About Market Intervention Scheme

 Market Intervention Scheme (MIS) is a price support mechanism implemented on the request of

State Governments for procurement of perishable and horticultural commodities in the event of a

fall in market prices.

 The Scheme is implemented when there is at least 10% increase in production or 10% decrease in

the rates over the previous normal year.

 Market Intervention Scheme works in a similar fashion to Minimum Support Pricebased


procurement mechanism for food grains, but is an adhoc mechanism.

 Its objective is to protect the growers of these horticultural/agricultural commodities from making

distress sale in the event of bumper crop during the peak arrival period when prices fall to very low

level. Implementation of MIS

 The Department of Agriculture & Cooperationis implementing the scheme.

 The MIS has been implemented in case of commodities like apples, kinnoo/malta, garlic, oranges,

galgal, grapes, mushrooms, clove, black pepper, pineapple, ginger, red-chillies, coriander seed etc.

 Under MIP, funds are not allocated to the States. Instead, central share of losses as per the

guidelines of MIP is released to the State Governments/UTs, for which MIP has been approved,

based on specific proposals received from them.

Que-10 Consider the following statements about National Anti-Profiteering Authority (NAPA) as sometimes seen in news: 1. It
tackles the unregulated deposit schemes in order to prohibit fraud companies from taking funds from the public. 2. NAPA has
the power to deregister an entity or business. 3. Directorate General of Safeguards in the Central Board of Excise & Customs
(CBEC) is part of the NAA. Which of the above options is/are correct? a) 1, 2 and 3 b) 1 and 2 only c) 2 and 3 only d) 3 and 4 only
Answer: C Solution: NAA functions towards the unfair profit-making activities by the trading community. It examines whether
input tax credits availed by any registered person or not. Enrich Your Learning: National Anti-Profiteering Authority To contain
illegal profiteering by the supplier, an authority was created to ensure that the reduction in rate of tax or benefits of input tax
credit is ultimately passed to the final consumer. Which Act? Section 171 of the Central Goods and Services Act? Functions and
Powers of NAA:

 The main function is to ensure that traders are not realizing unfair profit by charging high price from

consumers in the name of GST.

 The responsibility of NAA is to examine and check such profiteering activities and recommend

punitive actions including cancellation of Registration.

 The chairman, NAA along with 4 Technical members and with help of the Standing Committee,

Screening Committee in every state and the Directorate General of Safeguards in the Central Board

of Excise & Customs (CBEC), work together on the anti-profiteering front.

 NAA has the authority to deregister an entity or business if it fails to pass on the benefit of lower

taxes under GST to the customer.

 Deregistering a business will be the last course of action and extreme step against any violator

 NAA will recommend the return of undue profit which a business earned from not passing on

reduction and benefit of tax to consumers along with an 18 per cent interest. It can also impose a

penalty if it sees it necessary.

Day-11 Current MCQs:- Que-1 Consider the following statements regarding the National Highway (NH) project of Chardham: 1.
The project was launched by Uttarakhand government. 2. It comprises of development of national highways into a two-lane
stretch. 3. The Char Dham project will connect Badrinath Dham, Kedarnath Dham, Haridwar and Rishikesh. Which of the above
statements is/are correct? a) 1 and 2 only b) 2 and 3 only c) 1 and 3 only d) 1, 2 and 3 Answer: A Solution: Correct statement:
 The Char Dham project will connect Badrinath Dham, Kedarnath Dham, Gangotri, Yamunotri. Enrich Your Learning: Chardham
Highway project Chardham National Highway (NH) project comprises improvement as well as the development of 889 km
length of national highways into a two-lane stretch. The Char Dham project will connect Badrinath Dham, Kedarnath Dham,
Gangotri, Yamunotri, and part of the route leading to Kailash Mansarovar yatra. Seven stretches have been identified for
development under the Char Dham highway project. On NH-58,a stretch including High level bridges at Kaliasaur, from
Rishikesh to Rudraprayagwill be developed. On NH-58, stretch between Rudraprayag and Mana village will be developed. This
includes mitigation measures for chronic Lambagarh land slide zone. On NH-94, a stretch including a long tunnel near Chamba,
from Rishikesh to Dharasu. On NH-108, a stretch including mitigation measures of 3 land slide zones, between Dharasu and
Gangotri will be developed. On NH-94, stretch including tunnel near Sylkyara bend, from Dharasu to Yamunotri will be
developed. On NH-109, stretch including a viaduct near Sonprayag, from Rudraprayag to Gaurikund will be developed. On NH-
125, 150-km long stretch will be developed. Three by-pass roads will also be constructed on this stretch.
Why in news? The Supreme Court has modified the National Green Tribunal (NGT) order and cleared the decks for
construction of over 880-km all-weather roads under the Chardham highway project. The apex court has ordered to form a
panel to study environmental concerns related to the project. A high-powered committee (HPC) is being constituted by the
Ministry of Environment and Forest. Background The NGT, in September 2018, gave its conditional nod to the Chardham
highway project in view of larger public interest. It stated the project would cause irreversible damage to regional ecology.
After that NGO Citizen for Green Doon filed a petitioned in the apex court. Subsequently, NGT had directed to constitute a
panel headed by a former Uttarakhand High Court Judge to monitor the project. Chardham Yatra Chardham Yatra in
Uttarakhand ispilgrimage to four destinations – Yamunotri, Gangotri, Kedarnath and Badrinath. All of these four sites are
devoted to a specific deity. Gangotri is dedicated to the Goddess Ganga, Yamunotri is dedicated to the Goddess Yamuna,
Kedarnath is dedicated to Lord Shiva and is one of the 12 jyotirlingas, and Badrinath, which is also part of India’s Char Dham, is
dedicated to Lord Vishnu.

Que-2 Consider the following statements about International Energy Forum (IEF): 1. It accounts for 90% of global supply and
demand for oil and gas. 2. The IEF’s Ministerial Meetings are the world's largest gathering of Energy Ministers. 3. India has
never hosted a Ministerial Meeting of IEF. Which of the above statements is/are correct? a) 1 and 2 only b) 2 and 3 only c) 1 and
3 only d) 1, 2 and 3 Answer: A Solution: Correct statement:

 India has hosted 16thInternational Energy Forum (IEF) Ministerial Meeting from 10-12th April

2018 in New Delhi. Enrich Your Learning: International Energy Forum (IEF) The International Energy Forum (IEF) is an inter-
governmental, non-profit international organisation. It aims to foster greater mutual understanding and awareness of
common energy interestsamong its members. The 70 Member Countries of the Forum including India are signatories to the
IEF Charter, which outlines the framework of the global energy dialogue through this inter-governmental arrangement. IEF was
established in 1991 with headquarters in Riyadh, Saudi Arabia. The IEF is the neutral facilitator of informal, open, informed and
continuing global energy dialogue. The Forum's biennial Ministerial Meetings are the world's largest gathering of Energy
Ministers. The magnitude and diversity of this engagement is a testament to the position of the IEF as a neutral facilitator and
honest broker of solutions in the common interest. The IEF is unique in that participants not only include IEA and OPEC
countries, but also key international actors. Its membership covers all six continents accounts for 90% of global supply and
demand for oil and gas. The International Energy Forum also coordinates the Joint Organisations Data Initiative (JODI) which
is a concrete outcome of the global energy dialogue. News The International Energy Forum (IEF) is being held three days of
meetings in Riyadh in March, 2020. Key Fact: India has hosted 16thInternational Energy Forum (IEF) Ministerial Meeting from
10-12th April 2018 in New Delhi. The Fourth IEF-EU Energy Day on 18 February 2020 was celebrated at the IEF Headquarters in
Riyadh.

Que-3 Consider the following statements about the Mentor India Campaign: 1. The mission is correlated to Atal Innovation
Mission. 2. It is an initiative launched by Ministry of Human Resource Development. Which of the above statements is/are
correct? a) Only 1 b) Only 2 c) Both 1 and 2 d) Neither 1 nor 2 Answer: A Solution: Correct statement:

 It is an initiative launched by NITI Aayog in 2017. Enrich Your Learning: Mentor India Campaign Mentor India Campaign is a
strategic nation building initiative to engage leaders who can guide and mentor students at more than 900 Atal Tinkering Labs,
established across the country as a part of the Atal Innovation Mission. The initiative was launched by NITI Aayog in 2017. NITI
Aayog is looking for leaders who can spend anywhere between one to two hours every week in one or more such labs to
enable students experience, learn and practice future skills such as design and computational thinking. Objective: Mentor India
is aimed at maximizing the impact of Atal Tinkering Labs, possibly the biggest disruption in formal education globally. The idea
is to engage leaders who will nurture and guide students in the Atal Tinkering Labs. These labs are non-prescriptive by nature,
and mentors are expected to be enablers rather than instructors. Atal Tinkering Labs Atal Tinkering Labs are dedicated works
spaces where students from Class 6thto Class 12th learn innovation skills and develop ideas that will go on to transform India.
The labs are powered to acquaint students with state-of-the-art equipment such as 3D printers, robotics & electronics
development tools, Internet of things & sensors etc. Atal Innovation Mission Atal Innovation Mission is among one of the
flagship programs of the Government of India to promote innovation and entrepreneurship in the country to set up the Atal
Tinkering Labs across the country. It aims to give substantial boost to the innovation ecosystem and to catalyse the
entrepreneurial spirit in the country. The objective is to provide an innovation promotion platform involving academics,
researchers and entrepreneurs, drawing upon national and international experiences to foster a culture of innovation in India.
The Mission has set up such labs across India and aims to have 2,000 labs.

Que-4 Consider the following statements about Senna Spectabilis: 1. This plant species is considered as invasive in some
countries. 2. The plant is widely distributed to Indian forests, especially in Wayanad Wildlife Sanctuary. 3. Senna Spectabilis is a
toxic plant for humans. Which of the above statements is/are correct? a) 1 and 2 only b) 2 and 3 only c) 1 and 3 only d) 1, 2 and
3 Answer: A Solution: Correct statement:
 Senna Spectabilis is not a toxic plant for human. It has been commonly used in traditional

medicine for many years. Enrich Your Learning: Senna Spectabilis Senna Spectabilis is a small, rounded deciduous tree. It
belongs to Fabaceae family, and is grown as ornamental plant across tropical America. It can grow 7 – 18 metres tall; is
evergreen in climates with rain all year round, but can become deciduous in some regions, and produces yellow flowers.
Spectabilis has been commonly used in traditional medicine for many years. It possesses significant biological activity, such as
antibacterial, ant biofilm, antifungal and antioxidant properties. However, it is considered an environmental weed by the
Global Compendium of Weeds. It grows extremely fast, flowers and sets seed profusely, and re-sprouts immediately when cut.
It takes around 2-3 years for the plant to grow from a sapling to a tree. In about two years, the trees start to flower. One young
tree can produce around 6000 seeds every season. Survival rate of these seeds are 95-99% and they can survive for 8-9 years
in soil. Also it has been noticed that if you debark a Senna tree it suddenly starts flowering just for survival. In places like
Australia, Uganda, and Cuba this species is considered as invasive. Now it has made its way to Indian forests, especially South
India. Why in news? Nearly 3,000 sq km-stretch of the region, including the Wayanad Wildlife Sanctuary, North and South
Wayanad forest divisions and the adjacent Muthumalai, Bandipur and Nagarhole tiger reserves, have reported wild growth of
the invasive plant. The plant is widely distributed in the Muthanga and Tholpetty range of forests under the Wayanad Wildlife
Sanctuary. It would not allow the growth of other indigenous species of trees or even grass under its thick canopy. So, shrubs
and other native plants are being displaced, while no animals also seem to prefer feeding on the trees. Forest Department was
preparing to execute a project to eradicate the plant from the sanctuary.

Que-5 Survival International works for which of the following cause? a) Conservation of animal species b) Rights of people with
albinism c) Rights of tribal people d) Providing nutritional food to anaemic children Answer: C Solution: Correct statement:

 Survival International is the global movement for tribal peoples’ rights. Enrich Your Learning: Survival International Survival
International was founded in 1969 by a group of people appalled by the genocide of Amazon Indians. Survival has an
international secretariat based in London, UK. It is the global movement for tribal peoples’ rights. This is the only organization
that champions tribal peoples around the world. It helps them defend their lives, protect their lands and determine their own
futures. Survival works through projects and campaigns, education and publications. It works with around 80 different tribes
across the world. Aim Survival International is aimed to prevent the annihilation of tribal peoples and to give them a platform
to speak to the world so they can bear witness to the genocidal violence, slavery and racism they face on a daily basis.
Objectives: To help tribal peoples to exercise their rights to survival and self-determination. To ensure that the interests of
tribal peoples are properly presented in all decisions affecting their future. To secure for tribal peoples the ownership and use
of adequate land and other resources and to seek recognition of their rights over traditional land.

Que-6 Consider the following statements with reference to Global Findex Database: 1. It is the data set on how a country
implements digitalisation in financial transactions. 2. Use of mobile phones and the internet to conduct financial transactions is
added to the database of 2017. 3. The latest edition of the database was the third edition of it. Which of the above statements
is/are correct? a) 1 and 2 only b) 2 and 3 only c) 1 and 3 only d) 1, 2 and 3 Answer: B Solution: Correct statement:

 The Global Findex Database is the world’s most comprehensive data set on how adults save,

borrow, make payments, and manage risk. Enrich Your Learning: Global Findex Database The Global Findex Database is the
world’s most comprehensive data set on how adults save, borrow, make payments, and manage risk. Launched with funding
from the Bill & Melinda Gates Foundation, the database has been published every three years since 2011. The data are
collected in partnership with Gallup, Inc., through nationally representative surveys of adults in over 140 economies. Global
Findex database 2017 The 2017 edition was the latest of this database. It includes updated indicators on access to and use of
formal and informal financial services. And it adds new data on the use of financial technology (fintech), including the use of
mobile phones and the internet to conduct financial transactions. This third edition of the database points to advances in
digital technology that are key to achieving the World Bank goal of Universal Financial Access by 2020. Highlights of database
Between 2014 and 2017, the share of adults who have an account with a financial institution or through a mobile money
service rose globally from 62 percent to 69 percent. In developing economies, the share rose from 54 percent to 63 percent.
Yet, women in developing economies remain 9 percentage points less likely than men to have a bank account.

Que-7 Consider the following statements about the Missile Technology Control Regime (MTCR): 1. MTCR is a non-treaty
association of international governments. 2. The weapons under this regime does not include the biological weapons. 3. India
has joined the MTCR as a full member in 2018. Which of the above statements is/are correct? a) 1 and 2 only b) 2 and 3 only c)
1 and 3 only d) 1, 2 and 3 Answer: C Solution: Correct statement:
 MTCR includes unmanned delivery systems that could be used for chemical, biological, and

nuclear attacks. Enrich Your Learning: Missile Technology Control Regime (MTCR) The MTCR is an informal non-treaty
association of governments sharing common interests in the non- proliferation of missiles, unmanned air vehicles, and related
technologies. The regime consists of the Guidelines and an Equipment and Technology Annex. MTCR was established in April
1987, it aims to limit the spread of ballistic missiles and other unmanned delivery systems that could be used for chemical,
biological, and nuclear attacks. It has 35 member countries which include most of the world's key missile manufacturers. MTCR
urges to restrict their exports of missiles and related technologies capable of carrying a 500- kilogram payload at least 300
kilometres or delivering any type of weapon of mass destruction. Aim The regime goal is to limit the risks of proliferation of
weapons of mass destruction (i.e. nuclear, chemical and biological weapons) by controlling the transfers that could make a
contribution to delivery systems for such weapons. The MTCR has been credited with slowing or stopping several missile
programs by making it difficult for prospective buyers to get what they want or stigmatizing certain activities and programs. Yet,
the regime has its limitations. Iran, India, North Korea, and Pakistan continue to advance their missile programs. How the
MTCR Works? Each MTCR member is supposed to establish national export control policies for any type of weapon of mass
destruction and technologies that appear on the regime's Material and Technology Annex. The annex is divided into two
separate groupings of items, Category I and Category II. Category I includes complete missiles and rockets, major sub-systems,
and production facilities. Category II includes specialized materials, technologies, propellants, and sub-components for missiles
and rockets. The MTCR identifies five factors that members should take into account when evaluating a possible export of
controlled items:

 Whether the intended recipient is pursuing or has ambitions for acquiring weapons of mass

destruction.

 The purposes and capabilities of the intended recipient's missile and space programs.

 The potential contribution the proposed transfer could make to the intended recipient's

development of delivery systems for weapons of mass destruction.

 The credibility of the intended recipient's stated purpose for the purchase.

 Whether the potential transfer conflicts with any multilateral treaty. Because the regime is voluntary and the decision to
export is the sole responsibility of each member, the MTCR has no penalties for transfers of controlled items. Key Fact: India
has officially joined the Missile Technology Control Regime (MTCR) as a full member and became the 35 th member of the MTCR
in July, 2018.

Que-8 Consider the following statements: 1. The economy is growing at a decent rate but generating very few jobs is known as
jobless growth. 2. An industry showing promise of a rapid boom in its infancy stage is called sunrise industry. Which of the
above statements is/are correct? a) Only 1 b) Only 2 c) Both 1 and 2 d) Neither 1 nor 2 Answer: C Solution: Correct statement:

 All the given statements are correct. Enrich Your Learning: What Is Jobless Growth? Jobless growth means that while the
economy is growing at a decent rate (i.e. GDP) it is generating very few jobs, much less than the potential. Despite having
qualifications, many people remain unemployed. As the population of a country grows, people need to work in order to support
their families and themselves. An expanding economy is necessary to employ all those who seek work. Without sufficient
economic growth, people looking for work will be unable to find it. In a jobless growth economy, unemployment remains
stubbornly high even as the economy grows. This tends to happen when a relatively large number of people have lost their
jobs, and the ensuing recovery is insufficient to absorb the unemployed, under-employed, and those first entering the
workforce. Impacts of jobless growth on economy: The prospect of a jobless growth economy has ramifications for everyone.
An economy that is growing without showing concomitant growth in the number of jobs challenges investors, employees, and
industries to adapt to the new economic order. When growth is coupled with high unemployment, it means that the economy
is experiencing structural changes. Sunrise Sector Industry A sunrise industry is one that is new and is growing fast and is
expected to become important in the future. It is a burgeoning sector or business in its infancy stage showing promise of a
rapid boom. Sunrise industries are typically characterized byhigh growth rates, numerous start-ups,and an abundance of
venture capital funding. Examples of sunrise industries include hydrogen fuel production, petrochemical industry, food
processing industry, space tourism, and online encyclopaedias.
Que-9 Consider the following statements with reference to the Senkaku Islands: 1. They are located in the South China Sea. 2.
They were controlled by the US after World War II. 3. The sovereignty of these islands are claimed by China and Japan. Which of
the above statements is/are correct? a) 1 and 2 only b) 2 and 3 only c) 1 and 3 only d) 1, 2 and 3 Answer: B Solution: Correct
statement

 Senkaku Islands are located in the South China Sea Enrich Your Learning: Senkaku Islands

What is the dispute? The Senkakus islands in the East China Sea are claimed by China, Taiwan and Japan. The Japanese-
administered island chain, is located about 200km southwest of Japan’s Okinawa island and a similar distance northeast of
Taiwan. Japan annexed the archipelago following China’s defeat in the first Sino-Japanese war from 1894 to 1895. Yet the
islands were left out of the Treaty of San Francisco at the end of the second world war that returned to China most of the
territories previously occupied by Japan. Under the terms of Japan’s surrender, the island chain was controlled by the US until
1971, when it was returned to Japan along with Okinawa and other surrounding islands. Recently, the potential for oil reserves
in the area prompted China to reassert its territorial claims over the islands. Japan does not recognise China’s claims nor the
existence of a dispute over the islands’ sovereignty. Why are they so coveted? As they appear to have great promise as a future
oil province of the world. Abundant fishing resources can be found nearby, as can important shipping lanes used by Japan,
South Korea and China for energy imports. It also become a focal point of the broader rivalry between the two countries.

Que-10 Consider the following statements: 1. HCOC internationally regulate the area of ballistic missiles capable of carrying
weapons of mass destruction. 2. The HCOC does not ban ballistic missiles. 3. India is a non-signatory to HCOC. Which of the
above statements is/are correct? a) 1 and 2 only b) 2 and 3 only c) 1 and 3 only d) 1, 2 and 3 Answer: A Solution: Correct
statement:

 India joined on 1 June 2016, is the latest signatory of the HCOC. Enrich Your Learning: Hague Code of Conduct (HCOC) The
HCOC also known as the International Code of Conduct against Ballistic Missile Proliferation, was established on 25 November
2002
as an arrangement to prevent the proliferation of ballistic missiles. The Hague Code of Conduct against Ballistic Missile
Proliferation (HCOC) is the result of efforts of the international community to internationally regulate the area of ballistic
missiles capable of carrying weapons of mass destruction. The HCOC is the only multilateral code in the area of disarmament. It
is the only normative instrument to verify the spread of ballistic missiles. The HCOC does not ban ballistic missiles, but it does
call for restraint in their production, testing, and export. Since the signing and entering into force of the politically-binding HCOC
in November 2002 in The Hague (Netherlands) the number of signatories has increased from 93 to 143. As agreed by the
conference in The Hague, Austria serves as the Immediate Central Contact (Executive Secretariat) and therefore coordinates
the information exchange within the HCOC framework. India joined on 1 June 2016, is the latest signatory of the HCOC. Annual
Regular Meetings of Subscribing States to the HCOC (annual conferences) were held in Vienna. The 18th Regular Meeting took
place from 3 to 4 June 2019 under the chairmanship of Norway. The 19 th Regular Meeting will take place from 3 June to 4 June
2020
.

Day-12 Current MCQs:- Que-1

With reference to Financial Action Task Force (FATF), consider the following statements: 1. It is an inter-governmental body for
combating money laundering and terror financing. 2. It was established by the G-7 Summit. 3. Its secretariat is located in Hague,
Netherlands. 4. The Asian Development Bank has the observer status of FATF. Which of the statements given above is/are
correct? a) 1 and 3 only b) 1 and 2 only c) 3 and 4 only d) 1, 2 and 4 only Answer: D Solution:

 The FATF secretariat is located in PARIS. Enrich Your Learning: Financial Action Task Force (FATF)

 The Financial Action Task Force (FATF) is an inter-governmental body established in 1989 by the

Ministers of its Member jurisdictions.

 The objectives of the FATF are to set standards and promote effective implementation of legal,

regulatory and operational measures for combating money laundering, terrorist financing and other

related threats to the integrity of the international financial system.

 The FATF is therefore a “policy-making body” which works to generate the necessary political will to

bring about national legislative and regulatory reforms in these areas.

 The FATF has developed a series ofrecommendations that are recognised as the international

standard for combating of money laundering and the financing of terrorism and proliferation of

weapons of mass destruction.

 These standards the basis for a co-ordinated response to these threats to the integrity of the financial

system and help ensure a level playing field.

 The FATF monitors the progress of its members in implementing necessary measures, reviews money

laundering and terrorist financing techniques and counter-measures and promotes the adoption and

implementation of appropriate measures globally.

 The FATF's decision making body, the FATF Plenary, meets three times per year.

 The FATF currently comprises 37 member jurisdictions and 2 regional organisations, representing most

major financial centres in all parts of the globe.

 India is also a member of the FATP.

 The FATP secretariat is located at the OECD headquarters in PARIS. Funding for the FATF Secretariat and other services is
provided by the FATF annual budget to which members contribute.

Que-2 With reference to Advisory Board for Banking Frauds (ABBF), consider the following statements: 1. The Central Vigilance
Commission has constituted the ABBF. 2. The bank fraud cases over Rs. 100 crores will be examined by ABBF. 3. Its secretarial
services will be provided by the Reserve Bank of India. 4. The Central Bureau of Investigation may also refer any case to the
ABBF. Which of the statements given above is/are correct? a) 1, 2 and 4 only b) 2, 3 and 4 only c) 1, 3 and 4 only d) All of the
above Answer: C Solution:

 The bank fraud cases over Rs. 50 crores will be examined by ABBF. Enrich Your Learning: Advisory Board for Banking Frauds
(ABBF)
 The Central Vigilance Commission has constituted Advisory Board for Banking Frauds (ABBF).

 It has been constituted to examine bank fraud over Rs 50 crore and recommend action.

 The ABBF, formed in consultation with the RBI, would function as the first level of examination of all

large fraud casesbefore recommendations or references are made to the investigative agencies by

the respective public sector banks (PSBs).

 The four-member board's jurisdiction would be confined to those cases involving the level of officers

of General Manager and above in the PSB in respect of an allegation of a fraud in a borrowal account.

 Central Bureau of Investigation (CBI) may also refer any case or matter to the board where it has any

issue or difficulty or in technical matters with the PSB concerned.

 The tenure of the Chairman and members would be for a period of two years from August 21, 2019.

 Headquartered in Delhi, the Reserve Bank of India will provide required secretarial services, logistic

and analytical support along with the necessary funding to the board

 The board will also periodically carry out frauds analysis in the financial system and give inputs for

policy formulation related to the fraud to the RBI.

Que-3 Which of the following publishes the ‘Global Environment Outlook’ Report? a) International Union for Conservation of
Nature b) United Nations Conference on Environment and Development c) World Wildlife Fund d) United Nations
Intergovernmental Panel on Climate Change Answer: A Solution: Some of the reports and their publishing
institutions/organizations:

 Forest Principles:United Nations Conference on Environment and Development

 Living Planet Report: World Wildlife Fund

 World Climate Report:Greening Earth Society (Extra Information) Enrich Your Learning: Global Environment Outlook (GEO)
Report

 The Global Environment Outlook (GEO) global assessments provide an integrated analysis (e.g. social,

economic, environmental) of major trends that have shaped the environment.

 These reports provide world leaders with policy options to take immediate action to address

environmental issues by turning environmental discussions into practice.

 Using the integrated environmental assessment (IEA)methodology, UN Environment has produced

five GEO reports and a set of regional GEOs thus far, which have analyzed environmental state and

trends at the global and regional scales, described plausible outlooks for various time frames and

formulated policy options.

Que-4 Which of the following statements is/are correct regarding the Negative policy rate, as recently seen in news? 1. Under
this policy, the financial institutions don’t need to pay interest to central banks for parking excess reserves. 2. This policy boosts
the lending to business and consumers by banks. 3. This policy strengthens the country’s currency. Select the correct answer
using the codes given below: a) 1 only b) 2 and 3 only c) 2 only d) 1, 2 and 3 Answer: C Solution:  Under this policy, the
financial institutions have to pay interest to central banks for parking excess reserves.  The Negative Policy Rate weakens the
country’s currencyas it makes the currency of the country a less attractive for investment in comparison to other currencies.
Enrich Your Learning: How does negative interest rates policy work?  Some major central banks have resorted to
unconventional policy measures, including a negative rate policy.  Under a negative rate policy, financial institutions are
required to pay interest for parking excess reserves with the central bank. It means any surplus cash beyond that which
regulators say banks must keep on hand.  That way, central banks penalise financial institutions for holding on to cash in the
hope of prompting them to boost lending to businesses and consumers.  The euro area, Switzerland, Denmark, Sweden and
Japan have allowed rates to fall to slightly below zero.  The European Central Bank introduced negative rates in June 2014,
lowering its deposit rate to -0.1% to stimulate the economy. Advantages  They help weaken a country’s currency by making it
a less attractive investment than other currencies and gives boost to exports.  It also lower borrowing costs on a whole range
of instruments, meaning that businesses and households get even cheaper loans. Disadvantages  It narrows the margin that
financial institutions earn from lending.  The depositors can avoid being charged negative rates on their bank deposits by
choosing to store actual banknotes instead. Que-5 Consider the following statements with reference to Notifiable Disease in
India: 1. Notifiable diseases are any diseases that are required by law to be reported to government authorities. 2. Any failure
to report a notifiable disease is a criminal offence in India. 3. Rabies and botulism are not considered notifiable disease in India.
Which of the above statements is/are correct? a) 1 and 2 only b) 2 and 3 only c) 1 and 3 only d) 1, 2 and 3 Answer: A Solution:
Rabies and botulism are considered notifiable disease in India. Enrich Your Learning: Notifiable Disease:

 A notifiable disease is any disease that is required by law to be reported to government authorities.

 This reporting allows the authorities to monitor the disease, and provides early warning of possible

outbreaks.

 The World Health Organization’s International Health Regulations require disease reporting to the

WHO in order to help with its global surveillance and advisory role.

 Registered medical practitioners need to notify such diseases in a proper form within three days, or

notify verbally via phone within 24 hours depending on the urgency of the situation.

 The process helps the government keep track and formulate a plan for elimination and control.

 In less infectious conditions, it improves information about the burden and distribution of disease.

 The onus of notifying any disease and the implementation lies with the state government.

 Any failure to report a notifiable disease is a criminal offence and the state government can take

necessary actions against defaulters. Which diseases are notifiable disease?

 The Central government has notified several diseases such as cholera, diphtheria, encephalitis,

leprosy, meningitis, pertussis (whooping cough), plague, tuberculosis, AIDS, hepatitis, measles, yellow

fever, malaria dengue, etc.

Que-6 Consider the following statements with reference to Know India Programme: 1. It is a joint flagship programme of
Ministry of Tourism and Ministry of External Affairs. 2. It aims to familiarize Indian-origin youth across the world with
contemporary India. Which of the above statements is/are correct? a) Only 1 b) Only 2 c) Both 1 and 2 d) Neither 1 nor 2
Answer: B Solution:

 It is a joint flagship programme of Ministry of External Affairs. Enrich Your Learning:


- Know India Programme (KIP) is a flagship initiative for Diaspora engagement which familiarizes Indian-

origin youth (18-30 years) with their Indian roots and launched in 2004.

- It is 25-day orientation programme organized by Union Ministry of External Affairs (MEA) in

partnership with one or two States including visit to States for 10 days.

- The programme is open to youth of Indian origin (excluding non-resident Indians) with preference to

those from Girmitiya countries (Mauritius, Fiji, Suriname, Guyana, Trinidad &Tobago, Jamaica etc).

- The programme incorporates following aspects to familiarise Youths about Indian Society and culture:

o Understanding of India’s political system, economy, society, and developments in various

sectors etc.

o Visiting places of historical, cultural, religious importance.

o Familiarisation with art, music and culture of India.

o Visiting industrial sites.

o Visiting a village.
o Interaction with non-profit organizations.

o Meetings with senior leadership/officials in India.

o Visiting a select state in India for 10 days. Significance:

 It has a view to promote awareness on different facets of life in India and the progress made by the

country in various fields.

 KIP provide a unique forum for students & young professionals of Indian origin to visit India, share

their views, expectations & experiences and to develop closer bonds with the contemporary India.

Que-7 Which of the following group of states is habitat of Indian Star Tortoises in India? a) Gujarat, Rajasthan, Uttar Pradesh b)
Gujarat, Maharashtra, Uttarakhand c) Madhya Pradesh, Rajasthan, Uttar Pradesh d) Punjab, Rajasthan, Uttar Pradesh Answer:
A Solution & Enrich Your Learning: Indian star tortoise: Why in news?

 Recently, India’s proposal to upgrade the protection of star tortoises (Geochelone elegans) in CITES

(Convention on International Trade in Endangered Species on Wild Fauna and Flora) have been

approved.

 Star tortoises have been listed under Appendix I of CITES and will now enjoy the highest degree of

protection as there will be a complete international ban enforced on their trade. About star tortoise:

 Indian Star Tortoise is found in western India and extreme south eastern Pakistan (e.g., Gujarat,

Rajasthan, Uttar Pradesh and the Thar Desert in Pakistan), south eastern India (Karnataka, Kerala,

Tamil Nadu) and in Sri Lanka.

 Star tortoises from northern India are larger and darker than those from southern India, which tend

to be smaller and have more contrasting, star-like shell patterns.

 90% of trade of star tortoises occurs as part of the international pet market.

 The species is categorized as ‘vulnerable’ by the International Union of Conservation of Nature (IUCN)

and a decline greater than 30% was predicted by 2025 if the exploitation continued or expanded. Background of CITES:

 The convention resulted from a resolution adopted at a 1963 meeting of member countries of the

International Union for Conservation of Nature (IUCN).

 It was adopted in March 1973 to regulate worldwide commercial trade in wild animal and plant

species.

 The formal text of CITES was entered into force in 1975.

Que-8 Consider the following statements about Delimitation of Constituencies and Delimitation Commission: 1. Delimitation
commission consists of a retired Supreme Court judge, Chief Election Commissioner and the respective State Election
Commissioners. 2. The Parliament enacts a Delimitation Act after every Census under Article 82 of Indian constitution. 3. There
was no delimitation after the 1981 and 1991 censuses at the national level. Which of the above options is/are correct? a) 3 only
b) 2 and 3 only c) 1, 2 and 3 d) All of the above Answer: D Solution & Enrich Your Learning: Delimitation of Constituencies
Context:

 Bifurcation of Jammu and Kashmir state and the need to conduct delimitation exercise, to establish
electoral constituencies. J&K parliamentary seats remain as delimited on the basis of the 1971 Census. About Delimitation:

 Delimitation is the act of redrawing boundaries of Lok Sabha and state Assembly seats to represent

changes in population. These orders come into force on a date to be specified by the President of India.  Under Article 82, the
Parliament enacts a Delimitation Act after every Census. The Union government sets up a Delimitation Commission after the act
comes into force.  The commission consists of a retired Supreme Court judge, the Chief Election Commissioner and the
respective State Election Commissioners. The Constitution mandates that its orders are final and cannot be questioned before
any court. In case of difference of opinion, opinion of the majority in the commission prevails.  Delimitation Commissions have
been set up four times — 1952, 1963, 1973 and 2002 under the Acts of 1952, 1962, 1972 and 2002. There was no delimitation
after the 1981 and 1991 censuses. To allay the fears of Southern States, who have promoted family planning. Exercise was
conducted in the following situations:  These include statehood attained by Arunachal Pradesh and Mizoram in 1986, the
creation of a Legislative Assembly for the National Capital Territory of Delhi, and creation of new states such as

Que-9 Consider the following statements about Medical Tourism in India: 1. India has nearly 18% of the global medical tourism
market. 2. Since last few years, Patients from Afghanistan form the highest number of medical tourists to India. Which of the
above statements is/are correct? a) Only 1 b) Only 2 c) Both 1 and 2
 Since last few years, Patients from Bangladesh form the highest number of medical tourists to India. Enrich Your Learning:
Medical Tourism in India

 Also known as Wellness tourism/ Medical Tourism. It means people travelling abroad to receive

medical treatment. While for Indian, medical tourism helps earn valuable foreign exchange.

Data:

 The total number of patients visiting India in the year 2017 was 4.95 Lakh.

 Patients from Bangladesh, followed by, Afghanistan, Oman, Maldives, Uzbekistan, Sudan, Iraq and

Yemen visit India.

 India has nearly 18% of the global medical tourism market.

 India’s medical tourism industry could be worth $9 billion, and account for 20% of the global market

share by 2020. Factors: (Why India’s medical tourism industry could be worth $9 billion?)

 Confluence of highest quality and cost advantage is unique for India.

 Indian healthcare service is cheaper for foreigners than healthcare services provided in their own
countries.

 Getting Visas for medical purpose is much easier

 Availability of high end and best in the world facilities. Internationally accredited hospitals. skilled

Manpower, technologies to support medical diagnostics

Que-10 Consider the following statements about National Mission to Improve Learning Outcomes at the Elementary level
(NISHTHA): 1. NISHTHA initiative aims to improve and modernize the infrastructure of Elementary level educational institutions
in India. 2. Under this programme, teachers are trained to develop and strengthen personal-social qualities of students for their
holistic development. Which of the above statements is/are correct? a) Only 1 b) Only 2 c) Both 1 and 2 d) Neither 1 nor 2
Answer: B Solution:

 It aims to improve learning outcomes at the elementary level through an Integrated Teacher Training

Programme called NISHTHA. Enrich Your Learning: NISHTA Programme:

 Launched by The Department of School Education and Literacy. It Is the world's largest teachers'

training programme of its kind.

 To improve learning outcomes at the elementary level through an Integrated Teacher Training

Programme called NISHTHA under the Centrally Sponsored Scheme of Samagra Shiksha in 2019-20.

 Covering 42 lakh participants.

 Covers all teachers and Heads of Schools at the elementary level in all Government schools, faculty

members of State Councils of Educational Research and Training (SCERTs), District Institutes of

Education and Training (DIETs) as well as Block Resource Coordinators and Cluster Resource

Coordinators in all States and UTs.  The training Modules for NISHTHA have been developed through a consultative process
involving the

suggestions from the States and UTs and CBSE, KVS, NVS, School Principals and Non-Governmental

Organizations, such as Kaivalya Foundation, Tata Trust, Azim Premji Foundation and Aurobindo

Society. Aims to:  Build competencies among all the teachers and school principals at the elementary stage.  Motivate and
equip teachers to encourage and foster critical thinking in students.  The initiative is first of its kind wherein standardized
training modules are developed at national level

for all States and UTs. States and UTs can modify as per their needs, by keeping in view the core topics

and expected outcomes of NISHTHA.  Teachers are trained as first level counsellors to be alert and responsive to the social,
emotional and

psychological need of students.  Teachers are trained to use Art as pedagogy leading to increased creativity and innovation
among

students. Benefits and expected outcomes:  Teachers are trained to develop and strengthen personal-social qualities of
students for their holistic

development.  Creation of healthy and safe school environment.  Develop stress-free School Based Assessment focused on
development of learning competencies.  Teachers adopt Activity Based Learning and move away from rote learning to
competency based

learning.  Teachers and School Heads become aware of new initiatives in school education.  Transformation of the heads of
schools for providing academic and administrative leadership in the
schools for fostering new initiatives.

Day-13 Static MCQs:- Que-1 Consider the following statements: 1. Insurance claim is Capital receipt whereas interest received
on loan is a Revenue receipt. 2. Capital receipt increases the value of asset whereas Revenue receipt decreases the value of
asset. 3. Capital receipts are non- recurring whereas Revenue receipts are recurring. With reference to the differences between
Capital Receipt and Revenue Receipt, which of the above statements is/are correct? a) 1 only b) 2 and 3 only c) 1 and 3 only d)
1, 2 and 3 Answer: C Solution:

 Capital receipt decreases the value of asset whereas Revenue receipt increases or decreases the value

of asset or liability. Enrich Your Learning: Difference between revenue receipts and capital receipts

CAPITAL RECEIPTS REVENUE RECEIPTS It is the money brought to the business from non-operating sources like proceeds from
the sale of long-term assets, capital brought by the proprietor, sum received as a loan or from debenture holders etc.

These receipts are the result of firm’s routine activities

during the financial year, which includes sales,

commission, interest on investment.

They are part of the financing and investing activities rather than operating activities.

These receipts are a part of normal business operations.

It is Non-Recurring. It is Recurring i.e. occur again and again. It is shown in Balance sheet. It is shown in Income Statement.
Decreases the value of asset or increases the value of liability.

Increases or decreases the value of asset or liability.

The receipts can be generated from the Issue of shares, Debentures, government Grants, Insurance Claim or loan taken from a
bank.

The revenue receipts are generated from sale of inventory,

services rendered, interest received on loan, dividend,

rent received etc. The receipts are for long term. The receipts are for short term.

Que-2 Which of the following statements given below is/are correct with reference to commercial banks? 1. The commercial
banks accepts demand deposits only. 2. The spread is the difference between the time and demand deposit. Select the correct
answer using the codes given below: a) 1 only b) 2 only c) Both 1 and 2 d) Neither 1 nor 2 Answer: D Solution:

 The commercial banks accept demand deposits as well as time deposits.

 The ‘spread’ is the difference between the interest rate levied on loan and interest rate given on

deposits. The ‘spread’ is the profit of the bank. Enrich Your Learning: Commercial Banks

 Commercial banks are the other type of institutions which are a part of the money-creating system of

the economy.

 They accept deposits(time and demand deposits) from the public and lend out part of these funds

to those who want to borrow.

 The interest rate paid by the banks to depositors is lower than the rate charged from the borrowers.

 The difference between these two types of interest rates, called the ‘spread’ is the profit appropriated

by the bank.
 Commercial banks mediate between individuals or firms with excess funds and lend to those who need

funds. People with excess funds can keep their funds in the form of deposits in banks and those who

need funds, borrow funds in form of home loans, crop loans, etc. Commercial Banks are of three types:

1. Public Sector Banks

2. Private Sector Banks

3. Foreign Banks

Que-3 Consider the following statements: 1. The Industrial Policy Resolution of 1956 emphasized on establishment of heavy
industries. 2. The Industrial Policy of 1991 removed the rigid license system. Which of the statements given above is/are
correct? a) 1 only b) 2 only c) Both 1 and 2 d) Neither 1 nor 2 Answer: C Solution:Both the statements are correct. Enrich Your
Learning: Growth of Industry in India  Industry or the secondary sector of the economy is another important area of economic
activity. After independence, the government of India emphasized the role of industrialization in the country's economic
development in the long run. 1956  The blue print for industrial development was made through the Industrial Policy
Resolution (IPR) in 1956.  The 1956 policy emphasized on establishment of heavy industries with public sector taking the lead
in this area. It was justified on the ground that it will reduce the burden on agriculture, enable growth in the production of
consumer goods industries as well as small industries that are helpful for employment generation and achieving self-reliance.
1956 -1966 (second and third plans period)  Public sector contributed maximum to this growth but towards the end of 1960s,
investment in industries was reduced which adversely affected its growth rate. 1980’s  The trend was reversed and investment
in industries was increased by making the infrastructure base such as power, coal, rail much stronger. 1991  It was found that
the public sector undertakings (PSUs) were not performing upto expectation. Mismanagement and loss making was common in
PSU’s.  So in 1991 the government of Indian decided to encourage the role of private sector in industrial development and
removed the rigid licence system which is known as liberalization and allow international players to compete in the domestic
country as well as domestic players to explore foreign territories. Such a model of industrial development is called
Liberalization, Privatization and Globalization (LPG) model.  In the early years of 1990s there was significant growth in
industrialization due to increase in investment in infrastructure, reduction in excise duty, availability of finance etc. But towards
the end of 1990s the growth rate slowed down due to stiff competition from international companies, inadequate
infrastructure support etc.  The aim of taking all these steps was to strengthen the process of industrialization in the country.
 After the adoption of this new policy in 1991, there has been phases of growth followed by slowdown in the industrial
development process. Post 1991  In the beginning of the new millennium, between 2002-08 there was again some recovery
due to increase in saving rate from 23.5 percent in 2001-2 to 37.4 percent in 2007- 08.

 Even the competition from the foreign companies helped during this phase as the domestic companies

could create enough internal strength in term of quality control, finance and customer care etc. to

withstand the competition.

 However, after 2008-09 there was some slow-down in industrial growth due to rise in petroleum price,

interest rate and borrowings from abroad which has created lot of liabilities for the domestic

companies.

Que-4 Consider the following statements about agricultural sector during the British Rule: 1. Commercialization of agriculture
during the British rule mostly benefited small farmers. 2. Low of level of technology during the British rule was one of the
factors for low agricultural productivity. Which of the above statements is/are correct? a) Only 1 b) Only 2 c) Both 1 and 2 d)
Neither 1 nor 2 Answer: B Solution:

 Commercialization of agriculture during the British rule, benefitted large farmers. Enrich Your Learning: Agriculture during
the British Rule: About 85 per cent of the country’s population lived mostly in villages and derived livelihood directly or
indirectly from agriculture. Land settlement systems like Zamindari system, commercialisation of agriculture and many more
factors led to agricultural productivity being dismal.

Positives Negatives
 Expansion of the area under

cultivation.

 Commercialisation of agriculture

led to higher yield of cash crops.

Zamindari System:Introduced in Bengal Presidency. The zamindaris

had to deposit sums of revenue on a specific date, else would lose

their right

 Zamindars and the government did not focus on

improving the condition of agriculture.

Commercialisation of agriculture:

 Cash crops was limited to certain areas and practiced by

only large farmers.

 Also caused a shift in production from food crops to

commercial crops.

Others Problems:

 Low of level of technology

 Lack of irrigation systems

 Negligible use of fertilisers.

 Lack of investment in terracing, flood control, drainage

and desalinisation of soil.

Que-5 Consider the following statements about various types of Economic Systems: 1. In a mixed economic system, market
forces determine supply and demand. 2. In a capitalist economic system, desires of individuals are not given priority. 3. In a
socialist economy, the market produces goods and services in which it is competent. Which of the above options is/are NOT
correct? a) 1 and 2 only b) 2 and 3 only c) 1 and 3 only d) 1, 2 and 3 Answer: D Solution:

 In a capitalist economic system, market forces determine supply and demand.

 In a Socialist economy, desires of individuals are not given priority.

 In a mixed economic system, the market will produce goods and services in which it is competent Enrich Your Learning: Types
of Economic Systems Type of economic systems define which goods and services should be produced in a country, how should
these goods and services be produced and thirdly how should it be distributed among people.

Capitalist Economic System Socialist Economic System Mixed Economic System

 Also known as capitalism.

 Market forces determine

supply and demand.

 Produce goods and

services which are in


demand and can be

profitably sold.

 Goods are distributed

based on the ability of

people to buy goods and

services.

 India did not adopt this

model, due to high

poverty, a great majority

of people would be left

behind.

 Here the Government

decides what goods to be

produced and how to

distribute based on needs

of the society.

 Assumed that

Government knows what

is good for the people of

the country.

 Desires of individuals are

not given priority.

 This system has no private

property. Everything is

owned by the state.

 China is an example.

 Both the government and

market produce goods

together.

 Government will focus on

producing essential goods

and services

 The market will produce


goods and services in

which it is competent.

Que-6 Consider the following statements about Land Reforms in post-independence India: 1. Land reforms were successful in
Kerala and West Bengal due to the proactive role played by agrarian community of respective states. 2. Policy of land reform
was aimed at fixing the maximum size of land which could be owned by an individual. Which of the above statements is/are
NOTcorrect? a) Only 1 b) Only 2 c) Both 1 and 2 d) Neither 1 nor 2 Answer: C Solution:

 Land reforms were successful in Kerala and West Bengal due to the reason that their respective state

governments were in favour of the policy of land to the tiller.

 Policy of land reform was aimed to abolish intermediaries and make the tillers the owners of the land.

While the policy of Land ceiling aimed to fix the maximum size of land which could be owned by an

individual. Land Reforms in post-independence Presence of intermediaries like zamindars, jagirdars were the chief
characteristics of Indian Land tenure system. This resulted in large number of intermediaries, followed by concentration of land
ownership in a few hands, lack of improvements on farm. Ultimately leading to low productivity of agriculture sector in India
and forcing India to import food. Measures Taken: To ensure equity,

 Policy of land reforms was aimed to abolish intermediaries and make the tillers the owners of the land.

Followed by the policy of Land Ceiling which fixed the maximum size of land which could be owned by

an individual.

 Land reforms were successful in Kerala and West Bengal. Outcome:

 Lack of commitment at the political level, exploitation of legal loopholes, eviction of tenants,

landowners claimed to be self-cultivators among others. Big landlords buying time to register lands in

the name of close relatives by challenging the land ceiling legislation in courts.

Que-7 Consider the following statements about agriculture in Pre-British India era: 1. Butter was exported in abundance. 2.
Canal irrigation was available. 3. Tobacco was grown as a commercial crop. 4. Indian villages produced and consumed goods
and services within itself. Which of the above options is/are correct? a) 1, 2 and 4 only b) 1 and 3 only c) 3 and 4 only d) 1, 2, 3
and 4 Answer: D Solution & Enrich Your Learning: Agriculture During Pre-British India

 Indian economy, during the pre-British period, consisted of backward, isolated and self-sustaining

villages on the one hand and on the other hand, there were number of towns which were the seats of

administration, pilgrimage, commerce and handicrafts.

 Important crops included rice, wheat, bajra, jowar and minor cereals along with the commercial crops

like jute, raw cotton, groundnut, tobacco etc.

 India exported: In abundance, cottons and silks, rice, sugar and butter.

 Followed crop rotation practices. The agricultural implements were mostly primitive and simple which

included wooden plough, iron sickle, leather bag for drawing water etc. And use of natural organic

manures.

 Animal wealth of India, pigs, sheep, goats and fish. Irrigation through canals and navigation facilities

along the Ganges. But, farmers had to pay a high rate of taxes and faced high degree of exploitation.
Que-8 Consider the following statements: 1. Industrial revolution in Britain. 2. Development of irrigation. 3. Shift in production
from food crops to commercial cash crops. Which of the above factors led to the Commercialization of agriculture during
colonial period? a) 1 and 2 only b) 2 and 3 only c) 1 and 3 only d) 1, 2 and 3 Answer: D Solution & Enrich Your Learning:
Commercialisation of Agriculture

 The British Rule had multidimensional impacts on Indian economy. Among them, Commercialisation

of Indian Agriculture was a significant impact.

 Commercialisation of agriculture indicates production of various crops for sale. What led to Commercialisation of
Agriculture?

 Industrial revolution in Britain had raised the demand for agro-raw-materials, especially for raw

cotton, jute, sugarcane, groundnuts etc. for British industries.

 Higher prices offered for commercial crops to peasants.

 And development of irrigation intensified the commercialisation of agriculture Outcome:

 Shift in production from food crops to commercial cash crops.

 India turned into a supplier of raw material and a market for British products.

 Benefited only the large farmers.

 One of the major cause for famines causing large scale starvation deaths in India.

Que-9 Consider the following regions during colonial period: 1. Madras Presidency 2. Bengal 3. Rajasthan 4. Orissa The increase
in the share of workforce in agriculture, during colonial period, was noticed in which of the above regions? a) 1, 2 and 4 only b)
3 and 4 only c) 1 and 3 only d) 1, 2, 3 and 4 Answer: B Solution & Enrich Your Learning: Occupational Structure During the
colonial period, the occupational structure of India, i.e., distribution of working persons across different industries and sectors,
showed little sign of change.

 Agriculture accounted for largest share of workforce about 70-75 %. Manufacturing about 10% and

Services 15-20%.

 Variations across the country, Madras Presidency, Bombay and Bengal, a decline in dependence on

agriculture and increase in manufacturing and services sectors.

 Agriculture workforce increased in Odisha, Rajasthan and Punjab. Currently:

 Agriculture Workforce accounts for 43.86%, manufacturing 24.69% and services 31.45%.

Que-10 Consider the following statements about Industrial Sector during the British Rule: 1. Modern Industry in India began
from the second half of the nineteenth century. 2. The demand of traditional handicraft industry rose to counter the British
manufactured goods. 3. Public Sector was limited to railway, power generation and some other departmental undertakings.
Which of the above options is/are correct? a) 1, 2 and 3 b) 1 and 3 only c) 2 and 3 only d) 1 and 2 only Answer: B Solution:

 Decline in handicrafts industry led to high unemployment which deprived availability of locally made

good and indirectly created demand for British manufactured goods. Enrich Your Learning: Industrial Sector during the British
Rule: India did not develop a sound industrial base under the British Rule. Features:

1) The colonial government`s intention was 1) To make India an exporter of important raw material. 2)

Also make India a market for finished products. Ultimately benefitting Britain.

2) Decline in handicrafts industry led to high unemployment which deprived availability of locally made
good and indirectly created demand for British manufactured goods.

3) Modern Industry in India began from the second half of the nineteenth century. Progress was very

slow. Focus was on Cotton in Maharashtra and Gujarat and Jute Mills in Bengal.

4) Capital goods industries produce machine tools used for production of more articles. Like Iron and

Steel. During the British Rule, there were hardly any capital goods industries in India.

5) Industrial sector contribution to the Gross Domestic Product was very small.

6) Public Sector was limited to railway, power generation, communications, ports and some other

departmental undertakings.

Day-13 Current MCQs:- Que-1 Sharm El-Sheikh Declaration of 2018 is related to: a) Maritime security and cooperation among
Gulf countries. b) Permanent solution for Israel-Palestine crisis. c) Integrating biodiversity values in legislative and policy
frameworks. d) Formulating Air space rules. Answer: C Solution: Sharm El-Sheikh Declaration of 2018 is related to investing in
Biodiversity for People and Planet by Integrating biodiversity values in legislative and policy frameworks. Enrich Your Learning:
Sharm El-Sheikh Declaration:

 Sharm El-Sheikh Declaration is a declaration on investing in Biodiversity for People and Planet

by Integrating biodiversity values in legislative and policy frameworks.

 The High-level Segment (HLS) of the UN Biodiversity Conference brought together leaders from

national governments and international organizations during a two-day event immediately

preceding the 14th meeting of the Conference of the Parties (COP 14) to the Convention on

Biological Diversity (CBD) on November 2018.

 The Sharm El-Sheikh Declaration on ‘Investing in Biodiversity for People and Planet’ was

developed by Egypt following consultation with Parties. Commitment:

 Through the Declaration, the governments commit:

o To mainstream biodiversity,

o Integrating biodiversity values in legislative and policy frameworks, and development

and finance plans;

o Phasing out or reforming subsidies and other harmful incentives; strengthening

ecosystem-based approaches to climate change mitigation and adaptation;

o promoting sustainable consumption and production and a circular economy; and

o Facilitating access to and transfer of relevant technologies.

o Encourage integrated implementation of the 2030 Agenda for Sustainable

Development.

 The Declaration invites the UN General Assembly (UNGA) to convene a summit on biodiversity

before CBD COP 15 in 2020 to highlight the urgency of action at the highest levels in support of a

post-2020 global biodiversity framework.


Que-2 Consider the following statements with reference to Operation Greens: 1. The scheme aims to ensure year round
availability of TOP crops without price volatility. 2. The scheme is an initiative of Ministry of Agriculture and Farmers’ Welfare.
Which of the above statements is/are correct? a) 1 only b) 2 only c) Both 1 and 2 d) None Answer: A Solution: The scheme is an
initiative ofMinistry of Food Processing Industries. Enrich Your Learning: Operation Greens:

 In the budget speech of 2018-19, a new Scheme “Operation Greens” was announced on the line

of “ Operation Flood ”, with an outlay of Rs.500 crores to promote Farmer Producers

Organizations, agri-logistics, processing facilities and professional management.

 Operation Greens seeks to stabilize the supply of Tomato, Onion and Potato (TOP) crops and to

ensure availability of TOP crops throughout the country round the year without price volatility.

 The scheme is an initiative of the Ministry of Food Processing Industries.

 NAFED will be the Nodal Agency to implement price stabilisation measures. Objectives:

 Enhancing value realisation of TOP farmers by targeted interventions to strengthen TOP

production clusters and their FPOs, and linking/connecting them with the market.

 Price stabilisation for producers and consumers by proper production planning in the TOP

clusters and introduction of dual use varieties.

 Reduction in post-harvest losses by creation of farm gate infrastructure, development of

suitable agro-logistics, creation of appropriate storage capacity linking consumption centres.

 Increase in food processing capacities and value addition in TOP value chain with firm linkages

with production clusters.

 Setting up of a market intelligence network to collect and collate real time data on demand and

supply and price of TOP crops. Strategies: The scheme will have two-pronged strategy of Price stabilisation measures (for
short term) and Integrated value chain development projects (for long term). (I) Short term Price Stabilisation Measures:

 MoFPI will provide 50% of the subsidy on the following two components:

o Transportation of Tomato Onion Potato(TOP) Crops from production to storage;

o Hiring of appropriate storage facilities for TOP Crops;

(II) Long Term Integrated value chain development projects: i. Formation and Capacity Building of FPOs ii. Quality Production
iii. Post-harvest processing facilities - At Farm Level iv. Post-harvest processing facilities - At Main Processing Site v. Agri-
Logistics vi. Marketing/Consumption Points.

Que-3 Human Capital Index is released by which of the following organisations? a) United Nations Development Programme b)
World Bank c) International Monetary Fund d) International Labour Organisation Answer: B Solution:The first edition of Human
Capital Index (HCI) was released by World Bank in 2018. Enrich Your Learning: Human Capital Index (HCI): The first edition of
Human Capital Index (HCI) was released by World Bank in 2018.

 The HCI has been constructed for 157 countries.

 It seeks to measure the amount of human capital that a child born today can expect to attain

by age 18.

 The HCI index values are contended to convey the productivity of the next generation of
workers, compared to a benchmark of complete standard education and full health.

The HCI has three components:

 Survival, as measured by under-5 mortality rates;

 Expected years of Quality-Adjusted School which combines information on the quantity and

quality of education (quality is measured by harmonizing test scores from major international

student achievement testing programs and quantity from number of years of school that a child

can expect to obtain by age 18 given the prevailing pattern of enrolment rates across grades in

respective countries); and

 Health environment using two proxies of (a) adult survival rates and (b) the rate of stunting for

children under age 5. Difference between HDI and HCI:

 Human Development Index (HDI) is released by United Nations Development Programme.

 The HCI uses survival rates and stunting rate instead of life expectancy as measure of health,

and quality-adjusted learning instead of merely years of schooling as measure of education.

 HCI also excludes per capita income whereas the HDI uses it.

 Two significant changes from HDI are exclusion of income component and introduction of

quality adjustment in learning.

 Exclusion of income element and introduction of quality adjustment makes HCI far less

representative of Human Capital Development than the Index claims it to be. Major Highlights:

 For 56% of the world’s population, the HCI is at or below 0.50; and for 92% it is at or below 0.75.

 Hence only 8% of the population can expect to be 75% as productive as they could be.

 The HCI measures the Index outcomes for each country on a scale of 0 to 1.

 As expected the advanced economies such as North America and Europe mostly have HCI value

ofabove 0.75, while South Asia and Sub Saharan Africa have the lowest HCI among the regions.

 The HCI for India has been estimated at 0.44.

 A child born in India today will be only 44 % as productive when she grows up as she could be if

she enjoyed complete education and full health.

 The HCI in India for females is marginally better than that for males.

Que-4 Consider the following statements with reference to Commercial Paper: 1. CP are issued at a discount from face value
and reflects prevailing market interest rates. 2. Commercial paper is a long term instrument of investment. Which of the above
statements is/are correct? a) 1 only b) 2 only c) Both 1 and 2 d) None Answer: A Solution:Commercial papers can be issued for
maturities between a minimum of 7 days and a maximum of up to one year from the date of issue. Enrich Your Learning:
Commercial Paper:

 Commercial Paper (CP) is an unsecured money market instrument issued in the form of a

promissory note.
 It was introduced in India in 1990 with a view to enabling highly rated corporate borrowers to

diversify their sources of short-term borrowings and provides an additional instrument to

investors.

 Subsequently, primary dealers and all-India financial institutions were also permitted to issue

CP to enable them to meet their short-term funding requirements for their operations.

 Commercial papers can be issued for maturities between a minimum of 7 days and a maximum

of up to one year from the date of issue.

 CP are usually issued at a discount from face value and reflects prevailing market interest rates. Context:

 Aditya Birla Finance Ltd (ABFL), the NBFC arm of Aditya Birla Capital, became the first company

(November 2019) to list its commercial papers on the stock exchanges.

 The move comes after the exchanges -- the BSE and the NSE came out with a framework for

listing of commercial papers (CPs), in a bid to broaden investors' participation in such securities. Significance:

 Listing of CPs is expected to lead to efficient transmission of information regarding corporate

borrowings and liquidity positions to market participants.

 It will also contribute effectively towards development of the commercial paper market and is

expected to have a positive effect on the debt capital market.

Que-5 Consider the following statements with reference to Airports Economic Regulatory Authority of India (AERA): 1. AERA is a
statutory body constituted under the Airports Economic Regulatory Authority of India Act. 2. It determines the tariff for the
aeronautical services and amount of the Passengers Service Fee. Which of the above statements is/are correct? b) 2 only c)
Both 1 and 2 d) None Answer: C Solution:Both of the given statements are correct. Enrich Your Learning: Airports Economic
Regulatory Authority of India (AERA):

 AERA is a statutory body constituted under the Airports Economic Regulatory Authority of

India Act, 2008. Functions:

 To determine the tariff for the aeronautical services taking into consideration.

o The capital expenditure incurred and timely investment in improvement of airport

facilities,

o The cost for improving efficiency,

o Economic and viable operation of major airports,

o Revenue received from services other than the aeronautical services,

 To determine the amount of the Development Fees in respect of major airports.

 To determine the amount of the Passengers Service Fee levied under rule 88 of the Aircraft

Rules, 1937 made under the Aircraft Act, 1934.

 To monitor the set Performance Standards relating to quality, continuity and reliability of

service as may be specified by the Central Government or any authority authorized by it in this
behalf.

Que-6 Consider the following statements with reference to Rabies: 1. Rabies is a viral disease endemic to tropical countries. 2.
The highest burden of rabies lies on Asia and Africa. Which of the above statements is/are correct? a) 1 only b) 2 only c) Both 1
and 2 d) None Answer: B Solution: With the exception of Antarctica, rabies is endemic to all continents. Enrich Your Learning:
Rabies:

 Rabies is a zoonotic disease (a disease that is transmitted from animals to humans), caused by

the rabies virus, within the family Rhabdoviridae.

 Domestic dogs are the most common reservoir of the virus, with more than 99% of human

deaths caused by dog-mediated rabies.

 The virus is transmitted in the saliva of rabid animals.

 With the exception of Antarctica, rabies is endemic on all continents.

 95% of cases of rabies are reported in Asia and Africa.

 Rabies is a 100% vaccine-preventable disease.

 Elimination programs revolve around mass dog vaccination campaigns. National Rabies Control programme:

 National Rabies Control programmewas initiated in the 12th five-year plan.

 Under National Rabies Control programme(NRCP) funds are provided for training, surveillance,

laboratory strengthening etc.

 The funds are not provided for procurement of Anti-Rabies vaccine (ARV).

 However, for rabies vaccination all the states/UTs have been communicated to include Anti-

Rabies vaccine (ARV) and Anti Rabies serum (ARS) under essential drug list and to undertake

the procurement of ARV and ARS under National Free Drug service initiative under National

Health Mission.

 There is rabies elimination drive by 2030 globally.

 All the rabies vaccine manufacturers were also requested to ensure that the manufacturing of

ARV is carried out with full capacity and first preference may be accorded to meet domestic

requirements including Government Institutional supplies in the country.

 Drug Controller General of India (DCGI) has been directed to monitor the pharmaceutical firms

producing ARV for ensuring regular supply of ARV to the States.

Que-7 Operation Clean Art is related to which of the following wildlife species? a) Red Panda b) Mongoose c) Pangolin d) Tiger
Answer: B Solution: Operation Clean Art is related to Mongoose. Enrich Your Learning: Operation Clean art:

 Operation Clean Art was the first pan India operation to crackdown on the smuggling of

mongoose hair.

 It was conceived by Wildlife Crime Control Bureau (WCCB). Objective:

 Adult mongoose yields over 30-40 gm of long hair, from which 20-25 gm of “brush-making hair”
is recovered.

 Operation Clean Art was conceived by WCCB with the singular aim of ensuring that the

mongoose hair brush trade should be closed down across the country. About Mongoose:

 Mongoose are small animals native to southern Eurasia and mainland Africa.

 Mongooses are noted for their audacious attacks on highly venomous snakes such as king

cobras.

 In India, the mongoose is listed in Schedule II Part 2 of the Wildlife Protection Act, 1972 and

any smuggling or possession of its body part is a non-bailable offence.

 IUCN status of Indian Grey Mongoose:Least Concerned

Que-8 Consider the following statements with reference to Retrovirus: 1. Retrovirus is DNA virus. 2. Human immunodeficiency
virus is a retrovirus. Which of the above statements is/are correct? a) 1 only b) 2 only c) Both 1 and 2 d) None Answer: B
Solution:Retrovirus isRNA(Ribonucleic acid) virus. Enrich Your Learning: Retrovirus:

 A type of virus that uses RNA as its genetic material.

 After infecting a cell, a retrovirus uses an enzyme called reverse transcriptase to convert its

RNA into DNA.

 The retrovirus then integrates its viral DNA into the DNA of the host cell, which allows the

retrovirus to replicate.

 Human immunodeficiency virus (HIV), the virus that causes Acquired Immunodeficiency

Syndrome (AIDS), is a retrovirus.

Que-9 Match the following statements with reference to Joint Exercises conducted by Indian Army:

Exercises Countries

1. Garuda Shakti A. Mongolia

2. Nomadic Elephant B. Indonesia

3. Ekuverin C. Maldives

Select the correct answer using the codes given below: a) 1 -A, 2 -B, 3 -C b) 1 -B, 2 -A, 3 -C c) 1 -B, 2 -C, 3 -A d) 1 -C, 2 -A, 3 –B
Answer: B Solution:

Exercises Countries

1. Garuda Shakti A. Indonesia

2. Nomadic Elephant B. Mongolia

3. Ekuverin C. Maldives

Enrich Your Learning: Joint Exercises conducted by Army:


Que-10 Match the following statements with reference to Air Force exercises by Indian Air Force:

Air Force exercises Countries

1. Desert Eagle A. USA

2. Indra Dhanush B. UAE

3. Cope India C. UK

Select the correct answer using the codes given below: a) 1 -A, 2 -B, 3 -C b) 1 -B, 2 -A, 3 -C c) 1 -B, 2 -C, 3 -A d) 1 -C, 2 -A, 3 –B
Answer: C Solution:

Air Force exercises Countries

1. Desert Eagle A. UAE

2. Indra Dhanush B. UK

3. Cope India C. USA


Enrich Your Learning: Joint Exercises conducted by Air Force:

S.No. Country Exercise

1. Bangladesh Table Top

2. Israel Ex Blue Flag

3. Oman Ex Eastern Bridge

4. Russia Ex Indra

5. Singapore Joint Military Training

6. Thailand Ex SIAM Bharat

7. UAE Desert Eagle

8. UK Indra Dhanush

9. USA Cope India

Day-14 Static MCQs:- Que-1 Consider the following statements: 1. Insurance claim is Capital receipt whereas interest received
on loan is a Revenue receipt. 2. Capital receipt increases the value of asset whereas Revenue receipt decreases the value of
asset. 3. Capital receipts are non- recurring whereas Revenue receipts are recurring. With reference to the differences between
Capital Receipt and Revenue Receipt, which of the above statements is/are correct? a) 1 only b) 2 and 3 only c) 1 and 3 only d)
1, 2 and 3 Answer: C Solution:

 Capital receipt decreases the value of asset whereas Revenue receipt increases or decreases the value

of asset or liability. Enrich Your Learning: Difference between revenue receipts and capital receipts

CAPITAL RECEIPTS REVENUE RECEIPTS

It is the money brought to the business from non- operating sources like proceeds from the sale of long-term assets, capital
brought by the proprietor, sum received as a loan or from debenture holders etc.

These receipts are the result of firm’s routine

activities during the financial year, which includes

sales, commission, interest on investment.

They are part of the financing and investing activities rather than operating activities.

These receipts are a part of normal business

operations. It is Non-Recurring. It is Recurring i.e. occur again and again. It is shown in Balance sheet. It is shown in Income
Statement. Decreases the value of asset or increases the value of liability.

Increases or decreases the value of asset or liability.

The receipts can be generated from the Issue of shares, Debentures, government Grants, Insurance Claim or loan taken from a
bank.

The revenue receipts are generated from sale of

inventory, services rendered, interest received on

loan, dividend, rent received etc. The receipts are for long term. The receipts are for short term.
Que-2 Which of the following statements given below is/are correct with reference to commercial banks? 1. The commercial
banks accepts demand deposits only. 2. The spread is the difference between the time and demand deposit. Select the correct
answer using the codes given below: a) 1 only b) 2 only c) Both 1 and 2 d) Neither 1 nor 2 Answer: D Solution:

 The commercial banks accept demand deposits as well as time deposits.

 The ‘spread’ is the difference between the interest rate levied on loan and interest rate given on

deposits. The ‘spread’ is the profit of the bank. Enrich Your Learning: Commercial Banks

 Commercial banks are the other type of institutions which are a part of the money-creating system of

the economy.

 They accept deposits(time and demand deposits) from the public and lend out part of these funds

to those who want to borrow.

 The interest rate paid by the banks to depositors is lower than the rate charged from the borrowers.

 The difference between these two types of interest rates, called the ‘spread’ is the profit appropriated

by the bank.

 Commercial banks mediate between individuals or firms with excess funds and lend to those who need

funds. People with excess funds can keep their funds in the form of deposits in banks and those who

need funds, borrow funds in form of home loans, crop loans, etc. Commercial Banks are of three types:

4. Public Sector Banks

5. Private Sector Banks

6. Foreign Banks

Que-3 Consider the following statements: 1. The Industrial Policy Resolution of 1956 emphasized on establishment of heavy
industries. 2. The Industrial Policy of 1991 removed the rigid license system. Which of the statements given above is/are
correct? a) 1 only b) 2 only c) Both 1 and 2 d) Neither 1 nor 2 Answer: C Solution:Both the statements are correct. Enrich Your
Learning: Growth of Industry in India

 Industry or the secondary sector of the economy is another important area of economic activity. After

independence, the government of India emphasized the role of industrialization in the country's

economic development in the long run. 1956

 The blue print for industrial development was made through the Industrial Policy Resolution (IPR) in

1956.

 The 1956 policy emphasized on establishment of heavy industries with public sector taking the lead

in this area. It was justified on the ground that it will reduce the burden on agriculture, enable growth

in the production of consumer goods industries as well as small industries that are helpful for

employment generation and achieving self-reliance. 1956 -1966 (second and third plans period)

 Public sector contributed maximum to this growth but towards the end of 1960s, investment in

industries was reduced which adversely affected its growth rate. 1980’s

 The trend was reversed and investment in industries was increased by making the infrastructure base
such as power, coal, rail much stronger. 1991

 It was found that the public sector undertakings (PSUs) were not performing upto expectation.

Mismanagement and loss making was common in PSU’s.

 So in 1991 the government of Indian decided to encourage the role of private sector in industrial

development and removed the rigid licence system which is known as liberalization and allow

international players to compete in the domestic country as well as domestic players to explore foreign

territories. Such a model of industrial development is called Liberalization, Privatization and

Globalization (LPG) model.

 In the early years of 1990s there was significant growth in industrialization due to increase in

investment in infrastructure, reduction in excise duty, availability of finance etc. But towards the end

of 1990s the growth rate slowed down due to stiff competition from international companies,

inadequate infrastructure support etc.

 The aim of taking all these steps was to strengthen the process of industrialization in the country.

 After the adoption of this new policy in 1991, there has been phases of growth followed by slowdown

in the industrial development process. Post 1991

 In the beginning of the new millennium, between 2002-08 there was again some recovery due to

increase in saving rate from 23.5 percent in 2001-2 to 37.4 percent in 2007- 08.

 Even the competition from the foreign companies helped during this phase as the domestic companies

could create enough internal strength in term of quality control, finance and customer care etc. to

withstand the competition.

 However, after 2008-09 there was some slow-down in industrial growth due to rise in petroleum price,

interest rate and borrowings from abroad which has created lot of liabilities for the domestic

companies.

Que-4 Consider the following statements about agricultural sector during the British Rule: 1. Commercialization of agriculture
during the British rule mostly benefited small farmers. 2. Low of level of technology during the British rule was one of the
factors for low agricultural productivity. Which of the above statements is/are correct? a) Only 1 b) Only 2 c) Both 1 and 2 d)
Neither 1 nor 2 Answer: B Solution:

 Commercialization of agriculture during the British rule, benefitted large farmers. Enrich Your Learning: Agriculture during
the British Rule: About 85 per cent of the country’s population lived mostly in villages and derived livelihood directly or
indirectly from agriculture. Land settlement systems like Zamindari system, commercialisation of agriculture and many more
factors led to agricultural productivity being dismal.

Positives Negatives

 Expansion of the area under

cultivation.

 Commercialisation of
agriculture led to higher yield

of cash crops.

Zamindari System:Introduced in Bengal Presidency. The

zamindaris had to deposit sums of revenue on a specific date, else

would lose their right

 Zamindars and the government did not focus on

improving the condition of agriculture.

Commercialisation of agriculture:

 Cash crops was limited to certain areas and practiced

by only large farmers.

 Also caused a shift in production from food crops to

commercial crops.

Others Problems:

 Low of level of technology

 Lack of irrigation systems

 Negligible use of fertilisers.

 Lack of investment in terracing, flood control, drainage

and desalinisation of soil.

Que-5 Consider the following statements about various types of Economic Systems: 1. In a mixed economic system, market
forces determine supply and demand. 2. In a capitalist economic system, desires of individuals are not given priority. 3. In a
socialist economy, the market produces goods and services in which it is competent. Which of the above options is/are NOT
correct? a) 1 and 2 only b) 2 and 3 only c) 1 and 3 only d) 1, 2 and 3 Answer: D Solution:

 In a capitalist economic system, market forces determine supply and demand.

 In a Socialist economy, desires of individuals are not given priority.

 In a mixed economic system, the market will produce goods and services in which it is competent Enrich Your Learning: Types
of Economic Systems Type of economic systems define which goods and services should be produced in a country, how should
these goods and services be produced and thirdly how should it be distributed among people.

Capitalist Economic System Socialist Economic System Mixed Economic System

 Also known as capitalism.

 Market forces determine

supply and demand.

 Produce goods and services

which are in demand and can

be profitably sold.

 Goods are distributed based


on the ability of people to buy

goods and services.

 India did not adopt this

model, due to high poverty, a

great majority of people

would be left behind.

 Here the Government decides

what goods to be produced

and how to distribute based

on needs of the society.

 Assumed that Government

knows what is good for the

people of the country.

 Desires of individuals are not

given priority.

 This system has no private

property. Everything is owned

by the state.

 China is an example.

 Both the government and

market produce goods

together.

 Government will focus on

producing essential goods and

services

 The market will produce

goods and services in which it

is competent.

Que-6 Consider the following statements about Land Reforms in post-independence India: 1. Land reforms were successful in
Kerala and West Bengal due to the proactive role played by agrarian community of respective states. 2. Policy of land reform
was aimed at fixing the maximum size of land which could be owned by an individual. Which of the above statements is/are
NOTcorrect? a) Only 1 b) Only 2 c) Both 1 and 2 d) Neither 1 nor 2 Answer: C

 Land reforms were successful in Kerala and West Bengal due to the reason that their respective state

governments were in favour of the policy of land to the tiller.


 Policy of land reform was aimed to abolish intermediaries and make the tillers the owners of the land.

While the policy of Land ceiling aimed to fix the maximum size of land which could be owned by an

individual. Enrich Your Learning: Land Reforms in post-independence Presence of intermediaries like zamindars, jagirdars were
the chief characteristics of Indian Land tenure system. This resulted in large number of intermediaries, followed by
concentration of land ownership in a few hands, lack of improvements on farm. Ultimately leading to low productivity of
agriculture sector in India and forcing India to import food. Measures Taken: To ensure equity,

 Policy of land reforms was aimed to abolish intermediaries and make the tillers the owners of the land.

Followed by the policy of Land Ceiling which fixed the maximum size of land which could be owned by

an individual.

 Land reforms were successful in Kerala and West Bengal. Outcome:

 Lack of commitment at the political level, exploitation of legal loopholes, eviction of tenants,

landowners claimed to be self-cultivators among others. Big landlords buying time to register lands in

the name of close relatives by challenging the land ceiling legislation in courts.

Que-7 Consider the following statements about agriculture in Pre-British India era: 1. Butter was exported in abundance. 2.
Canal irrigation was available. 3. Tobacco was grown as a commercial crop. 4. Indian villages produced and consumed goods
and services within itself. Which of the above options is/are correct? a) 1, 2 and 4 only b) 1 and 3 only c) 3 and 4 only d) 1, 2, 3
and 4 Answer: D Solution & Enrich Your Learning: Agriculture During Pre-British India

 Indian economy, during the pre-British period, consisted of backward, isolated and self-sustaining

villages on the one hand and on the other hand, there were number of towns which were the seats of

administration, pilgrimage, commerce and handicrafts.

 Important crops included rice, wheat, bajra, jowar and minor cereals along with the commercial crops

like jute, raw cotton, groundnut, tobacco etc.

 India exported: In abundance, cottons and silks, rice, sugar and butter.

 Followed crop rotation practices. The agricultural implements were mostly primitive and simple which

included wooden plough, iron sickle, leather bag for drawing water etc. And use of natural organic

manures.

 Animal wealth of India, pigs, sheep, goats and fish. Irrigation through canals and navigation facilities

along the Ganges. But, farmers had to pay a high rate of taxes and faced high degree of exploitation.

Que-8 Consider the following statements: 1. Industrial revolution in Britain. 2. Development of irrigation. 3. Shift in production
from food crops to commercial cash crops. Which of the above factors led to the Commercialization of agriculture during
colonial period? a) 1 and 2 only b) 2 and 3 only c) 1 and 3 only d) 1, 2 and 3 Answer: D Solution & Enrich Your Learning:
Commercialisation of Agriculture

 The British Rule had multidimensional impacts on Indian economy. Among them, Commercialisation

of Indian Agriculture was a significant impact.

 Commercialisation of agriculture indicates production of various crops for sale. What led to Commercialisation of
Agriculture?

 Industrial revolution in Britain had raised the demand for agro-raw-materials, especially for raw
cotton, jute, sugarcane, groundnuts etc. for British industries.

 Higher prices offered for commercial crops to peasants.

 And development of irrigation intensified the commercialisation of agriculture Outcome:

 Shift in production from food crops to commercial cash crops.

 India turned into a supplier of raw material and a market for British products.

 Benefited only the large farmers.

 One of the major cause for famines causing large scale starvation deaths in India.

Que-9 Consider the following regions during colonial period: 1. Madras Presidency 2. Bengal 3. Rajasthan 4. Orissa The increase
in the share of workforce in agriculture, during colonial period, was noticed in which of the above regions? a) 1, 2 and 4 only b)
3 and 4 only c) 1 and 3 only d) 1, 2, 3 and 4 Answer: B Solution & Enrich Your Learning: Occupational Structure During the
colonial period, the occupational structure of India, i.e., distribution of working persons across different industries and sectors,
showed little sign of change.

 Agriculture accounted for largest share of workforce about 70-75 %. Manufacturing about 10% and

Services 15-20%.

 Variations across the country, Madras Presidency, Bombay and Bengal, a decline in dependence on

agriculture and increase in manufacturing and services sectors.

 Agriculture workforce increased in Odisha, Rajasthan and Punjab. Currently:

 Agriculture Workforce accounts for 43.86%, manufacturing 24.69% and services 31.45%.

Que-10 Consider the following statements about Industrial Sector during the British Rule: 1. Modern Industry in India began
from the second half of the nineteenth century. 2. The demand of traditional handicraft industry rose to counter the British
manufactured goods. 3. Public Sector was limited to railway, power generation and some other departmental undertakings.
Which of the above options is/are correct? a) 1, 2 and 3 b) 1 and 3 only c) 2 and 3 only d) 1 and 2 only Answer: B Solution:

 Decline in handicrafts industry led to high unemployment which deprived availability of locally made

good and indirectly created demand for British manufactured goods. Enrich Your Learning: Industrial Sector during the British
Rule: India did not develop a sound industrial base under the British Rule. Features:

7) The colonial government`s intention was 1) To make India an exporter of important raw material. 2)

Also make India a market for finished products. Ultimately benefitting Britain.

8) Decline in handicrafts industry led to high unemployment which deprived availability of locally made

good and indirectly created demand for British manufactured goods.

9) Modern Industry in India began from the second half of the nineteenth century. Progress was very

slow. Focus was on Cotton in Maharashtra and Gujarat and Jute Mills in Bengal.

10) Capital goods industries produce machine tools used for production of more articles. Like Iron and

Steel. During the British Rule, there were hardly any capital goods industries in India.

11) Industrial sector contribution to the Gross Domestic Product was very small.

12) Public Sector was limited to railway, power generation, communications, ports and some other

departmental undertakings. Day-14 Current MCQs:-


Que-1 With reference to National Clean Air Programme (NCAP), which of the statements given is/are correct? 1. The
programme targets for 20% - 30% reduction in PM 2.5 concentration only. 2. The NCAP will be a mid-term five-year action plan.
3. The certification agencies will be set up for monitoring air quality equipment. Select the correct answer using the codes given
below: a) 1 and 3 only b) 1 and 2 only c) 2 only d) 2 and 3 only Answer: D Solution:

 The programme targets for 20% - 30% reduction in PM 2.5 concentration and PM 10 concentration as

well. Enrich Your Learning: National Clean Air Programme (NCAP)

 It is launched by Ministry of Environment, Forest and Climate Change in January 2019.

 The tentative national level target of 20%–30% reduction of PM2.5 and PM10 concentration by 2024

is proposed under the NCAP taking 2017 as the base year for the comparison of concentration.

 It objective is comprehensive mitigation actions for prevention, control and abatement of air pollution

besides augmenting the air quality monitoring network across the country and strengthening the

awareness and capacity building activities.

 The crux of the programme is collaborative and participatory approach involving relevant Central

Ministries, State Governments, local bodies and other Stakeholders with focus on all sources of

pollution.

 The NCAP will be a mid-term, five-year action plan with 2019 as the first year.

 There will be use of the Smart Cities program to launch the NCAP in the 43 smart cities falling in the

list of the 102 non-attainment cities.

 The other features of NCAP included increasing number of monitoring stations in the country including

rural monitoring stations, technology support, emphasis on awareness and capacity building

initiatives, setting up of certification agencies for monitoring equipment, source apportionment

studies, emphasis on enforcement, specific sectoral interventions etc. Current development

 The Ministry of Environment, Forest and Climate Change recently designated the Council of Scientific

and Industrial Research-National Physical Laboratory (CSIR-NPL) as a national agency that shall be

responsible for carrying out certification for instruments and equipments for monitoring emissions

and ambient air.

 Most of the instrument used for Continuous Ambient Air Quality Monitoring Stations (CAQMS) and

Continuous Emission Monitoring Systems (CEMS) are imported and therefore, their certification is not

based on the climatic and environmental conditions that are prevalent across India.

Que-2 Consider the following statements about International Telecommunication Union (ITU): 1. It is headquartered at New
York, USA. 2. It allocates global radio spectrum as well as satellite orbits. 3. It is the oldest global international organisation.
Which of the above options is/are correct? a) 1, 2 and 3 b) 1 and 2 only c) 2 and 3 only d) 1 and 3 only Answer: C Solution:

 Headquartered in Geneva, Switzerland Enrich Your Learning: Why in news? India has been elected as a member of the
International Telecommunications Union Council (ITU) for another 4-year term - from 2019 to 2022. India has been an active
member of the ITU since 1869. And a regular member of the ITU Council since 1952. About International Telecommunication
Union:
 Founded in 1865 and headquartered in Geneva, Switzerland. The oldest global international

organisation. And a specialised agency of United Nations.

 The ITU has 193 member states who elect representatives to the council.

 It works to facilitate international connectivity in communications networks, ITU allocates global radio

spectrum and satellite orbits, develops the technical standards that ensure networks and technologies

seamlessly interconnect, and strives to improve access to ICTs to underserved communities

worldwide. ITU has three main areas of activity organized in ‘Sectors’.

 Radio-communications: Radio communication Sector (ITU-R) coordinates this vast and growing range

of radio communication services, as well as the international management of the radio-frequency

spectrum and satellite orbits.

 Standardisation: ITU standards (called Recommendations) are fundamental to the operation of ICT

networks which allow communication devices to operate optimally.

 Development: ITU's Telecommunication Development Sector (ITU-D) helps companies enter emerging

markets, policy advocacy and capacity building programs in emerging and developing countries.

Que-3 Consider the following statements about Ek Bharat-Vijayee Bharat: 1. It is 3-month long programme. 2. It is organized to
commemorate 50th year of Vivekananda rock memorial. Which of the above statements is/are correct? a) Only 1 b) Only 2 c)
Both 1 and 2 d) Neither 1 nor 2 Answer: B Solution:

 It is a yearlong programme. Enrich Your Learning: Ek Bharat-Vijayee Bharat programme

 It is a yearlong mass contact programme organised to commemorate 50th year of Vivekananda rock

memorial. To make people aware of the inspiring story of Vivekananda Rock Memorial (Located at the

tri-junction of Indian ocean, Bay of Bengal and Arabian sea in Kanyakumari) and the activities of

Vivekananda Kendra.

 Will involve 20000 workers across the country of Vivekananda Kendra.

Que-4 ‘E-course on Vulnerability Atlas of India’ is an initiative of? a) Ministry of Home Affairs b) Ministry of Housing and Urban
Poverty Alleviation c) Ministry of Earth Sciences d) Ministry of Environment, Forest and Climate Change Answer: B Solution &
Enrich Your Learning:

E-course on ‘Vulnerability Atlas of India’

 Launched by Ministry of Housing and Urban Affairs.

 Offered jointly by the School of Planning and Architecture and Building Materials and Technology

Promotion Council (BMTPC).

 To create awareness and understanding about natural hazards and help in identification of regions

with high vulnerability such as earthquakes, and landslides, floods, etc.

 Also specifies district-wise level of damage risks to the existing housing stock. Benefits:

 The course could be a tool for effective & efficient disaster mitigation & management in the field of
Architecture, Civil Engineering, Urban & Regional Planning, Housing & Infrastructure Planning,

Construction Engineering & Management and Building & Materials Research

Que-5 Which of the following statements is/are correct with reference to Hepatitis C? 1. It is a disease affecting nervous system
and activities of spinal cord. 2. Its virus is a blood borne virus. 3. It has no effective vaccine. Select the correct answer using the
codes given below: a) 1 and 2 only b) 2 and 3 only c) 1 and 3 only d) 1, 2 and 3 Answer: B Solution:

 It is a disease affecting human Liver. Enrich Your Learning: Hepatitis C

 Hepatitis C is a liver disease caused by the hepatitis C virus (HCV): the virus can cause both acute and

chronic hepatitis, ranging in severity from a mild illness lasting a few weeks to a serious, lifelong illness.

 Hepatitis C is a major cause of liver cancer.

 The hepatitis C virus is a blood borne virus: the most common modes of infection are through

exposure to small quantities of blood. This may happen through injection drug use, unsafe injection

practices, unsafe health care, transfusion of unscreened blood and blood products, and sexual

practices that lead to exposure to blood.

 Globally, an estimated 71 million people have chronic hepatitis C virus infection.

 A significant number of those who are chronically infected will develop cirrhosis or liver cancer.

 WHO estimated that in 2016, approximately 399 000 people died from hepatitis C, mostly from

cirrhosis and hepatocellular carcinoma (primary liver cancer).

 Antiviral medicines can cure more than 95% of persons with hepatitis C infection, thereby reducing

the risk of death from cirrhosis and liver cancer, but access to diagnosis and treatment is low.

There is currently no effective vaccine against hepatitis C; however, research in this area is ongoing

Que-6 Consider the following statements about Central Drugs Standard Control Organisation (CDSCO): 1. It grants licenses to
specialized categories of critical drugs such as blood and blood products. 2. It is responsible for conducting clinical trials. 3. It
establishes Viral Research & Diagnostic Laboratories (VRDL) for diagnosis of the viral and other infectious diseases. Which of the
above options is/are correct? a) 1, 2 and 3 b) 1 and 3 only c) 2 and 3 only d) 1 and 2 only Answer: D Solution:

 Indian Council of Medical Research (ICMR) establishes Viral Research & Diagnostic Laboratories (VRDL)

for diagnosis of the viral and other infectious diseases. Enrich Your Learning: About Central Drugs Standard Control
Organisation (CDSCO)

 Under the Ministry of Health & Family Welfare, headquartered in New Delhi.

 Created to ensure uniform implementation of the provisions of the Act & Rules.

 To ensure safety, efficacy and quality of the medical product manufactured, imported and distributed

in the country. Mandate:

 CDSCO is responsible for approval of Drugs, Conduct of Clinical Trials,

 laying down the standards for Drugs, control over the quality of imported Drugs in the country and

coordination of the activities of State Drug Control Organizations by providing expert advice.

 CDSCO along with state regulators, is jointly responsible for grant of licenses of certain specialized
categories of critical Drugs such as blood and blood products, I. V. Fluids, Vaccine and Sera.

Que-7 Reducing Food Loss and Waste Report, as recently seen in news, was released by the: a) World Bank b) Food and
Agriculture Organisation c) World Resources Institute d) International Fund for Agricultural Development Answer: C Solution &
Enrich Your Learning: Reducing Food Loss and Waste report released by the World Resources Institute. Key Points mentioned
in the report:

 Nearly one-third of the food that is produced each year goes uneaten, costing the global economy

over $940 billion.

 The uneaten food is responsible for emitting about 8 per cent of planet-warming greenhouse gases

into the atmosphere

 Roots and tubers are the food group that face the maximum wastage, at over 62% for 2007. Fruits and

vegetables follow, with over 41%. (According to Food and Agriculture Organisation data)

 By weight, fruits and vegetables make up the largest share of total annual food loss and waste.

 Moreover, food loss and waste have many negative economic and environmental impacts. Que-8 Lassen, Sunway TaihuLight
and Summit are: a) New weapons systems developed by Israel b) Spacecrafts to be launched by NASA and European Space
Agency c) Supercomputers in the world d) An integrated software package for virtual reality and augmented reality software
Answer: C Solution & Enrich Your Learning: Context:

 The mission will enable India to leapfrog to the league of world-class computing power nations.

 A Rs 4500 crore mission to connect national academic and R&D institutions with a grid of 73 high

performance computing facilities.

 Will be networked and support the National Knowledge Network. And includes the development of

human resources for High Performance Computing (HPC).

 The Mission supports the government's vision of 'Digital India' and 'Make in India' initiatives.
 Funding from Ministry of Science and Technology and Ministry of Electronics and Information

Technology. And will be implemented by the Department of Science and Technology and Department

of Electronics and Information Technology (DeitY) through Centre for Development of Advanced

Computing (C-DAC) and Indian Institute of Science (IISc), Bangalore.

 Eventurally, the supercomputers will be designed and manufactured by CDAC in India.

 PARAM Shavak is one such machine that has been deployed to provide training.

Que-9 Consider the following statements about the Special Data Dissemination Standard (SDDS): 1. It is a World Bank standard
to guide member countries in the dissemination of national statistics to the public. 2. Recently, India has failed to comply with
Special Data Dissemination Standard. 3. Its subscription is mandatory for all member countries of World Bank. Which of the
above statements is/are correct? a) Only 1 b) Only 2 c) Both 1 and 3 d) 1, 2 and 3 Answer: B Solution:

 SDDS is an International Monetary Fund standard to guide member countries in the dissemination of

national statistics to the public.

 Subscription to the SDDS is voluntary for member countries of International Monetary Fund (IMF). Enrich Your Learning:
Special Data Dissemination Standard (SDDS):

 The Special Data Dissemination Standard (SDDS) was established by the IMF in 1996 to guide members

access to international capital markets in providing key economic and financial data to the public.

 There are currently 66 members including India in SDDS.

 The Reserve Bank of India is signatories of SDDS.

 The purpose of the SDDS is to guide member countries in the dissemination of comprehensive, timely,

accessible, and reliable economic and financial statistical data in the context of increasing economic

and financial integration.

 The SDDS identifies four dimensions of data dissemination:

1. Data: coverage, periodicity (frequency), and timeliness.

2. Access by the public.

3. Integrity of the disseminated data.

4. Quality of the disseminated data.

 It provides a National Summary Data Page (NSDP) to provide quick access to a single comprehensive

source of economic and financial data in the subscriber's data.

 The subscribing member needs to be committed to observing the standard and provide information

about its data and data dissemination practices.

 SDDS subscription indicates that a country meets the test of “good statistical citizenship.”

 Member countries’ subscription to the SDDS is voluntary.

 In 2018, India failed to comply with multiple requirements prescribed in the Special Data

Dissemination Standard (SDDS) due to technical glitches.


Que-10 The Janaushadhi Suvidha, as mentioned in news, is a: a) Oxo- Biodegradable sanitary napkins b) Oxo- Biodegradable
plastics c) Oxo- Biodegradable Band-Aid d) Oxo- Biodegradable syringes Answer: A Solution:

 Janaushadhi Suvidha is an Oxo-Biodegradable Sanitary Napkinsavailable at Janaushadhi Kendras

under the Pradhan Mantri Bhartiya Janaushadhi Pariyojana. Enrich Your Learning: Janaushadhi Suvidha

 It is an Oxo-Biodegradable Sanitary Napkins which will be available to women at Rs.1 per napkin from

now on. These are subsidised napkins.

 The facility of these cheap sanitary napkins was started on the Eve of World Environment Day 2018

under the Pradhan Mantri Bhartiya Janaushadhi Pariyojana (PMBJP).

 This has been launched by the Ministry of Chemical and Fertilizers.

 Jan Aushadhi Suvidha comes with a special additive, which makes it biodegradable when it comes in

contact with oxygen after being discarded.

 They are sold at Pradhan Mantri Bhartiya Janaushadhi Kendras. Key Facts

 According to the National Family Health Survey 2015-16, about 58 per cent of women aged between

15 to 24 years use locally prepared napkins, sanitary napkins and tampons.

 About 78 percent women in urban areas use hygienic methods of protection during menstrual period;

only 48 percent women in rural areas have access to clean sanitary napkins.

You might also like